Quiz Biochemistry

100
Biochemistry 14Mar2009 DO NOT DISTRIBUTE - 1 - 1 – Metabolic Fuels and Dietary Components 1) A 23-year-old presents for a consult about weight reduction. She works as a corporate sales consultant for popular women’s clothing. At 5 feet 7 inches tall, she weights 99 lbs and is very afraid she could weight over 100 lbs. Her daily exercise routine includes one hour of jogging every morning and one hour of walking every evening. Which of the following is the most likely problem afflicting this patient? a) Anorexia b) Depression c) Obesity d) Malnutrition e) Hypothyroidism 2.1) One calorie is equal to: a) 4.18 Calories b) 4.18 kilocalories c) 4.18 kilojoules d) 4.18 Joules e) 4.18 calories 2.2) Physicians tend to use units of calories, in part because that is what their patients use and understand. Food “calories” are actually Calories. How many calories are in a Calorie? a) 10,000 b) 1,000 c) 100 d) 10 e) 1 3.1) When fat is completely oxidized from CO2 to H20, how much fuel energy is generated (in kcal/g)? a) 2 b) 4 c) 7 d) 9 e) 11 3.2) An analysis of a patient’s diet shows they ate 90g carbohydrate, 10g protein, and 10g fat each day. Approximately how many calories did this patient consume in one day? a) 440 b) 470 c) 490 d) 740 e) 940 3.3) Glucose, which is the predominant sugar in human blood, is a: a) Monosaccharide b) Disaccharide c) Polysaccharide d) Triacylglycerol e) Linear chain of amino acids

description

Quiz Biochemistry

Transcript of Quiz Biochemistry

Page 1: Quiz Biochemistry

Biochemistry 14Mar2009

DO NOT DISTRIBUTE - 1 -

1 – Metabolic Fuels and Dietary Components

1) A 23-year-old presents for a consult about weight reduction. She works as a corporate

sales consultant for popular women’s clothing. At 5 feet 7 inches tall, she weights 99 lbs

and is very afraid she could weight over 100 lbs. Her daily exercise routine includes one

hour of jogging every morning and one hour of walking every evening. Which of the

following is the most likely problem afflicting this patient?

a) Anorexia

b) Depression

c) Obesity

d) Malnutrition

e) Hypothyroidism

2.1) One calorie is equal to:

a) 4.18 Calories

b) 4.18 kilocalories

c) 4.18 kilojoules

d) 4.18 Joules

e) 4.18 calories

2.2) Physicians tend to use units of calories, in part because that is what their patients use

and understand. Food “calories” are actually Calories. How many calories are in a

Calorie?

a) 10,000

b) 1,000

c) 100

d) 10

e) 1

3.1) When fat is completely oxidized from CO2 to H20, how much fuel energy is

generated (in kcal/g)?

a) 2

b) 4

c) 7

d) 9

e) 11

3.2) An analysis of a patient’s diet shows they ate 90g carbohydrate, 10g protein, and 10g

fat each day. Approximately how many calories did this patient consume in one day?

a) 440

b) 470

c) 490

d) 740

e) 940

3.3) Glucose, which is the predominant sugar in human blood, is a:

a) Monosaccharide

b) Disaccharide

c) Polysaccharide

d) Triacylglycerol

e) Linear chain of amino acids

Page 2: Quiz Biochemistry

Biochemistry 14Mar2009

DO NOT DISTRIBUTE - 2 -

3.4) Which of the following accounts for the difference in energy between fats and

carbohydrates/proteins?

a) Nitrogen

b) Water

c) Hydrogen

d) Ammonia

e) Oxygen

4) Other than adipose tissue, which of the following is a primary fuel store in the body?

a) Protein

b) Glucose

c) Alcohol

d) Glycogen

e) Triacylglycerol

5.1) For Body Mass Ratio (BMR), what is the assumed value of kcal/day/kg body

weight?

a) 1

b) 12

c) 24

d) 36

e) 72

5.2) What is the BMR for an “average” 70kg person?

a) 70 * 1 = 70 kcal/day

b) 70 * 12 = 840 kcal/day

c) 70 * 24 = 1,680 kcal/day

d) 70 * 36 = 2,520 kcal/day

e) 70 * 72 = 5,040 kcal/day

6) Which of the following is NOT a significant factor that affects BMR?

a) Pregnancy

b) Thyroid status

c) Body temperature

d) Age

e) Race

7.1) Which of the following Daily Energy Expenditure (DEE) hourly activity factors is

used for moderate activity, such as hoeing?

a) 1.0

b) 1.5

c) 2.5

d) 5.0

e) 7.0

7.2) For a 60-year-old female, the BMR equation is 10.5W+596. Which of the following

equations would be used for this 50kg patient if she spent her entire day lying in bed?

a) (10.5 * 50 + 596) * 1.0

b) 10.5 * 50 * 1.0 + 596

c) (10.5 * 50 + 596) * 1.5

d) 10.5 * 50 * 1.5 + 596

e) 10.5 * 50 + 596 + 1.0

Page 3: Quiz Biochemistry

Biochemistry 14Mar2009

DO NOT DISTRIBUTE - 3 -

8) Which of the following equations would be used for the previous patient if she spent 8

hours a day standing and 16 hours a day lying in bed?

a) 10.5 * 50 * 1.0 * 1.5 + 596

b) 10.5 * 50 * 1.0 + 596

c) 10.5 * 50 * (8 *1.5 + 16 * 1.0) + 596

d) (10.5 * 50 + 596) * (8 *1.0 + 16 * 1.5)

e) (10.5 * 50 + 596) * ((8 *1.5 + 16 * 1.0) / 24)

9) What is the DEE hourly activity factor for a 70kg, 25-year-old male who works heavy

labor 6 hours a day, does moderate exercise for 3 hours, sleeps for 7 hours, and sits

upright in a chair watching sports TV for the rest of the time?

a) (6 * 7.0 + 3 * 2.5 + 7 * 5.0 + 8 * 1.5)

b) (6 * 7.0 + 3 * 5.0 + 7 * 1.0 + 8 * 1.5) / 24

c) (6 * 7.0 * 3 * 5.0 * 7 * 1.0 * 8 * 1.5) / 24

d) (6 * 5.0 + 3 * 2.5 + 7 * 1.0 + 8 * 1.5) / 24

e) (6 * 5.0 * 3 * 2.5 * 7 * 1.0 * 8 * 1.5) / 24

10) What is the overall DEE activity factor for a 24-year-old male who does 1.5 hours of

heavy activity (weight lifting and Judo), 1 hour of moderate activity (racquetball), 1.5

hours of light activity, 16 hours of very light activity, and 4 hours of sleep?

a) (1.5 * 7.0 * 1 * 5.0 * 1.5 * 2.5 * 16 * 1.5 * 4 * 1.0)

b) (1.5 * 7.0 + 1 * 5.0 + 1.5 * 2.5 + 16 * 2.0 + 4 * 1.0) / 24

c) (1.5 * 7.0 + 1 * 5.0 + 1.5 * 2.5 + 16 * 1.5 + 4 * 1.0) / 24

d) (1.5 * 5.0 + 1 * 3.0 + 1.5 * 2.5 + 16 * 1.5 + 4 * 1.0) / 24

e) (1.5 * 7.0 + 1 * 3.0 + 1.5 * 2.5 + 16 * 1.5 + 4 * 1.0) / 24

11.1) Which of the following equations is the English equivalent for Body Mass Index

(BMI), from the metric equation weight/height^2 (kg/m^2)?

a) (weight * 2.2) / (height^2)

b) (weight) / (height^2) + 704

c) (weight) / (height^2) + 504

d) (weight * 704) / (height^2)

e) (weight * 504) / (height^2)

11.2) What is the BMI for a 100kg person that is 175cm tall?

a) 100 / (0.175^2)

b) 100 / (1.75^2)

c) 100 / (17.5^2)

d) 220 / (0.175^2)

e) 220 / (1.75^2)

12) If someone’s daily energy expenditure matched their daily caloric intake and they

worked out two full hours a day, what type of caloric balance would they have?

a) Positive (gain weight)

b) Neutral

c) Negative (lose weight)

13) For a 35-year-old male, the BMR equation is 11.6W+879. If a 120kg 35-year-old

male patient did light work for one hour a day, very light work for fifteen hours a day,

and slept for the rest of the day, what would his DEE be?

a) (11.6 * 120 + 879) * (8 * 1.0 + 15 * 1.5 + 1 * 2.5) / 24 = 3122

b) (11.6 * 120 + 879) + (8 * 1.0 + 15 * 1.5 + 1 * 2.5) / 24 = 2272

Page 4: Quiz Biochemistry

Biochemistry 14Mar2009

DO NOT DISTRIBUTE - 4 -

c) (11.6 * 120 + 879) * (8 * 1.0 + 15 * 2.5 + 1 * 5.0) / 24 = 4778

d) (11.6 * 240 + 879) * (8 * 1.0 + 15 * 1.5 + 1 * 2.5) / 24 = 5036

e) (11.6 * 240 + 879) + (8 * 1.0 + 15 * 1.5 + 1 * 2.5) / 24 = 3664

14) If the above 120kg 35-year-old male patient ate an average of 585g of carbohydrates,

150g of protein, and 95g of fat each day, what is their kcal/day?

a) 585 * 4 + 150 * 9 + 95 * 4 = 4070

b) 585 * 7 + 150 * 4 + 95 * 4 = 5075

c) 585 * 9 + 150 * 4 + 95 * 4 = 6245

d) 585 * 4 + 150 * 4 + 95 * 7 = 3605

e) 585 * 4 + 150 * 4 + 95 * 9 = 3795

15) Which of the following is true for the above 120kg 35-year-old male patient

regarding their caloric balance?

a) Positive

b) Neutral

c) Negative

16) If a patient consumes an excess of 100 kcal/day (about 8 ounces of full-sugar soda)

for an entire year, about how much weight would he/she gain?

a) ((100 * 365) / 4) / 1000 / 2.2 = 4lbs

b) ((100 * 365) / 9) / 1000 / 2.2 = 2lbs

c) ((100 * 365) / 4) / 1000 * 2.2 = 20lbs

d) ((100 * 365) / 9) / 1000 * 2.2 = 9lbs

e) ((100 * 365) / 9) / 100 * 2.2 = 90lbs

17) Which of the following dietary requirements is usually divided into electrolytes?

a) Water

b) Vitamins

c) Minerals

d) Essential fatty acids

e) Proteins

18) Which of the following dietary requirements is involved in nitrogen balance?

a) Essential fatty acids

b) Vitamins

c) Minerals

d) Carbohydrates

e) Proteins

19.1) Which of the following is NOT a nutritionally essential fatty acid (derived from

plant oils or derived from fish oils)?

a) !-linoleic acid

b) !-linolenic acid

c) "-lipoic acid

d) Eicosapentanoic acid (EPA)

e) Docosahexaenoic acid (DHA)

19.2) Which of the following is NOT a nutritionally essential amino acid?

a) Lysine

b) Leucine

c) Threonine

d) Asparagine

Page 5: Quiz Biochemistry

Biochemistry 14Mar2009

DO NOT DISTRIBUTE - 5 -

e) Valine

19.3) Which of the following is NOT a nutritionally essential amino acid?

a) Histidine

b) Proline

c) Methionine

d) Phenylalanine

e) Tryptophan

20.1) A deficiency in which of the following would lead to dark spots on the skin, spongy

gums, and sunken eyes (scurvy)?

a) Choline

b) Biotin

c) Folate

d) Thiamin

e) Vitamin C

20.2) A deficiency in which of the following would lead to a bright red tongue and

pigmented rash in areas exposed to sunlight (pellagra)?

a) Riboflavin

b) Vitamin C

c) Niacin

d) Folate

e) Vitamin B12

20.3) A deficiency in which of the following would lead to edema, anorexia, weight loss,

apathy, and confusion (beri-beri)?

a) Vitamin D

b) Vitamin K

c) Niacin

d) Thiamin

e) Vitamin B12

20.4) A deficiency in which of the following would lead to inadequate bone

mineralization (rickets)?

a) Vitamin C

b) Vitamin A

c) Vitamin D

d) Vitamin E

e) Vitamin B6

20.5) A deficiency in which of the following would lead to keratinization of the GI

epithelium, night blindness, and failure to produce tears (xerophthalmia)?

a) Vitamin C

b) Vitamin A

c) Vitamin D

d) Vitamin E

e) Vitamin B6

21) Which of the following is NOT a fat-soluble vitamin?

a) Vitamin A

b) Vitamin C

c) Vitamin D

Page 6: Quiz Biochemistry

Biochemistry 14Mar2009

DO NOT DISTRIBUTE - 6 -

d) Vitamin E

e) Vitamin K

22) The caloric content per gram of fuel:

a) Is higher for carbohydrates than triacylglycerols

b) Is higher for protein than for fat

c) Is proportionate to the amount of oxygen in fuel

d) Is the amount of energy that can be obtained from oxidation of the fuel

e) Is higher for children than adults

2 – The Fed or Absorptive State

1.1) Which of the following is the major carbohydrate of the human diet?

a) Glucose

b) Fructose

c) Maltose

d) Cellobiose

e) Starch

1.2) Which of the following is the major digestive enzyme that works in the mouth and

small intestine?

a) Lactase

b) Sucrase

c) Amylase

d) Lipase

e) Direct absorption by epithelial cells

1.3) Proteins are cleaved to amino acids by proteases. Which of the following proteolytic

enzymes from the pancreas does NOT act in the lumen of the small intestine?

a) Pepsin

b) Trypsin

c) Chymotrypsin

d) Elastase

e) Carboxypeptidase

1.4) Bile salts, which emulsify fats, are created in the ____ and stored in the ____.

a) Pancreas; Liver

b) Pancreas; Gallbladder

c) Liver; Pancreas

d) Liver; Gallbladder

e) Liver; Intestine

1.5) Elevated levels of chylomicrons were measured in the blood of a patient. What

dietary therapy would be most helpful in lowering chylomicron levels?

a) Decreased intake of calories

b) Decreased intake of fat

c) Decreased intake of cholesterol

d) Decreased intake of starch

e) Decreased intake of sugar

1.6) What is absorbed into the intestinal epithelial cells from micro-droplet micelles?

a) Triacylglycerides

b) Amylase and triacylglycerides

Page 7: Quiz Biochemistry

Biochemistry 14Mar2009

DO NOT DISTRIBUTE - 7 -

c) Free fatty acids and monosaccharides

d) Free fatty acids and chylomicrons

e) Free fatty acids and 2-monoacylglycerols

2) In response to a high carbohydrate meal, insulin is ____ and glucagon is ____.

a) Released; Release

b) Released; Suppressed

c) Suppressed; Suppressed

d) Suppressed; Released

3.1) Which of the following is NOT a liver product of glucose after a meal?

a) Glucagon

b) Glycogen

c) Triacylglycerols

d) CO2

e) ATP

3.2) From which of the following processes do red blood cells get all of their energy?

a) TCA cycle

b) Fatty acid oxidation

c) Electron transport chain

d) Anerobic glycolysis

e) Oxydative phosphorylation

3.3) Which of the following does NOT depend on insulin levels being high to use glucose

as energy?

a) Liver

b) Muscle

c) Brain

d) Adipose tissue

4.1) During the fed state, where are VLDLs created?

a) Intestine

b) Brain

c) RBCs

d) Liver

e) Muscle

4.2) What is the major function of chylomicrons and VLDLs produced in the fed state?

a) Transport of glucose

b) Transport of triacylglycerols

c) Storage of glucose (glycogen)

d) Storage of triacylglycerols (adipose)

e) Making triacylglycerols non-polar

5.1) Amino acids derived from digestion of dietary protein:

a) Provide nitrogen for synthesis of nonessential amino acids in the liver

b) Can be converted to glucose in most tissues

c) Cannot be converted to adipose tissue fat

d) Release nitrogen that is converted to urea in skeletal muscle

e) Are generally converted to body proteins or excreted in the urine

5.2) After digestion of a mixed meal:

a) Insulin stimulates the transport of glucose into the brain

Page 8: Quiz Biochemistry

Biochemistry 14Mar2009

DO NOT DISTRIBUTE - 8 -

b) Dietary triacylglycerols are transported in the portal vein to the liver

c) Glucagon is released from the pancreas

d) Monosaccharides are transported to adipose tissue via the lymphatic system

e) Glucose levels increase in the blood

3 – Fasting

1.1) As glucose levels drop and the fasting state is entered (basal metabolic state), insulin

levels ____ and glucagon levels ____.

a) Decrease; Increase

b) Increase; Decrease

c) Decrease; Decrease

d) Increase; Increase

1.2) Which of the following plays a crucial role in converting glycogen stores to glucose

during the basal metabolic state?

a) Brain

b) Kidney

c) Adipose

d) Muscle

e) Liver

1.3) What is the red blood cell product of glycolysis?

a) Lactate

b) Glycerol

c) Fatty acids

d) Amino acids

e) Triacylglycerols

1.4) During prolonged fasting (starvation), ketone body use is ____ and fatty acid use is

____.

a) Increased; Decreased

b) Increased; Increased

c) Decreased; Decreased

d) Decreased; Increased

2) Which of the following ketone bodies produced in the liver is NOT used as an energy

source by the muscle and kidney?

a) !-Hydroxybutyrate

b) Acetoacetate

c) Acetone

3.1) What supplies amino acids for gluconeogenesis during the basal metabolic state?

a) Liver

b) Adipose

c) Brain

d) RBCs

e) Muscle

3.2) What is the body’s main fuel during fasting?

a) Amino acids

b) Fatty acids

c) Ketone bodies

Page 9: Quiz Biochemistry

Biochemistry 14Mar2009

DO NOT DISTRIBUTE - 9 -

d) Lactate

e) Urea

3.3) By 24 hours after a meal:

a) Gluconeogenesis in the liver is the major source of blood glucose

b) Muscle glycolysis provides glucose to the blood

c) Muscles convert amino acids to blood glucose

d) Fatty acids released from adipose tissue provide carbon for synthesis of glucose

e) Ketone bodies provide carbon for gluconeogenesis

4) During the fasting state, an increase in glucagon would ____ lipase activity and the

release of fatty acids and would generate ____ acetyl CoA and ketone bodies.

a) Decrease; Less

b) Decrease; More

c) Increase; Less

d) Increase; More

5) Which of the following is NOT a carbon source for gluconeogenesis?

a) Lactate

b) Glycerol

c) Acetyl CoA

d) Amino acids

6) Which of the following increases during prolonged fasting?

a) Brain use of ketone bodies

b) Muscle use of ketone bodies

c) Liver gluconeogenesis

d) Muscle protein degradation

e) Liver production of urea

4 – Water, Acids, Bases and Buffers

1.1) What percentage of total body weight is water?

a) 20%

b) 40%

c) 60%

d) 80%

e) 100%

1.2) What type of bonding accounts for water’s ability to act as a solvent for polar

organic molecules and inorganic salts?

a) Ionic

b) Covalent

c) Metallic

d) Hydrogen

e) Disulfide

1.3) Which of the following is true about water?

a) Its heat of fusion is high

b) Its head of vaporization is low

c) Its resistance to temperature change is low

d) Its thermal conductivity is low

e) Its heat capacity is low

Page 10: Quiz Biochemistry

Biochemistry 14Mar2009

DO NOT DISTRIBUTE - 10 -

2) If a patient’s blood was hyperosmolar from an electrolyte imbalance, what would

happen to the tissue surrounding the vessels?

a) Fluid would move into the tissue from the vessels, expanding the tissue

b) Fluid would move into the tissue from the vessels, lysing the tissue cells

c) No fluid movement would occur

d) Fluid would move into the vessels from the tissue, dehydrating the tissue

e) Fluid would move into the vessels from the tissue, expanding the tissue

3.1) Which of the following is important in maintaining pH homeostasis?

a) Acid

b) Base

c) pKa

d) pKb

e) Buffer

3.2) Since pKa is the reciprocal of the dissociation constant, a stronger acid would yield:

a) An increase in pKa

b) No change in pKa

c) A decrease in pKa

4.1) Based on the Henderson-Hasselbalch equation (pH = pKa + log ([A-] / [HA])), at

what point is the pH of a weak acid equal to its pKa?

a) 0% dissociation

b) 25% dissociation

c) 50% dissociation

d) 75% dissociation

e) 100% dissociation

4.2) A decrease of blood pH from 7.5 to 6.5 would be accompanied by which of the

following changes in ion concentration?

a) A 10-fold increase in hydrogen ion concentration

b) A 10-fold increase in hydroxyl ion concentration

c) An increase in hydrogen ion concentration by a factor of 7.5/6.5

d) A decrease in hydrogen ion concentration by a factor of 6.5/7.5

e) A shift in concentration of buffer anions, with no change in hydrogen ion

concentration

5) Say the pH of the stomach is 3.0, urine is 5.0, plasma is 7.4, and intestines are basic to

counteract the acidity of the stomach. Using the Henderson-Hasselbalch equation, which

of the following is true?

a) Weak acids will absorb slowly in the stomach

b) Weak bases would not absorb in the intestines

c) Weak bases would absorb in the stomach

d) Weak bases will absorb very quickly in urine

e) Weak bases and acids would absorb in plasma

6) A drug that has a pKa = 3.5 is taken orally. It is known that this drug is absorbed

through the stomach when it has no charge (which is its acid form). What percentage of

this drug will have no charge in the stomach if the pH = 2?

a) 100% - [A-] / [HA] = 96.84%

b) [A-] / [HA] = 3.16%

c) 100% - [HA] / [A-] = 68.38%

Page 11: Quiz Biochemistry

Biochemistry 14Mar2009

DO NOT DISTRIBUTE - 11 -

d) [HA] / [A-] = 31.62%

7) Which of the following is NOT true for the titration curve of a weak acid, such as

acetic acid?

a) pKa is found at the pH when 0.5 equivalents of OH- are added

b) At pKa, addition of OH- will increase pH

c) At pKa, removal of OH- will decrease pH

d) pKa is found at pH = 7.0

e) A plateau in pH from 0.25 to 0.75 equivalents OH- would indicate buffering

8.1) CO2 entering the red blood cells is converted to:

a) Phosphate

b) Hemoglobin

c) Carbonic acid

d) Bicarbonate

e) Histidine

8.2) The hypothalamus contains the respiratory center and is sensitive to changes in pH.

If pH begins to drop, what will happen?

a) Breathing will increase

b) Breathing will decrease

c) Breathing will not change

d) Breathing will stop

e) Breathing will become more rhythmic

8.3) As bicarbonate leaves the red blood cell, ____ ions enters as an exchange.

a) Phosphate

b) Sodium

c) Potassium

d) Magnesium

e) Chloride

8.4) Which of the following physiologic/pathologic conditions is most likely to result in

an alkalosis, provided that the body could not fully compensate?

a) Production of lactic acid by muscles during exercise

b) Production of ketone bodies by a patient with diabetes mellitus

c) Repeated vomiting of stomach contents

d) Diarrhea with loss of the bicarbonate anions secreted into the intestine

e) An infection resulting in a fever and hypercatabolism

5 – Structures of the Major Compounds of the Body

1) Which of the following groups could NOT be oxidized or reduced?

a) Carboxylic acid

b) Ketone

c) Ether

d) Amino

e) Ester

Match the following functional groups with their condensation products:

2.1) Acid + Alcohol a) Ester

2.2) Acid + Amine b) Thioester

2.3) Acid + Acid c) Amide

Page 12: Quiz Biochemistry

Biochemistry 14Mar2009

DO NOT DISTRIBUTE - 12 -

2.4) Acid + Sulfhydryl d) Phosphoester

2.5) Phosphoric acid + Alcohol e) Anhydride

3) Which of the following is NOT true:

a) Oxidation involves losing hydrogen

b) Reduction involves gaining electrons

c) Oxidation involves losing oxygen

d) Reduction involves gaining hydrogen

e) Oxidation involves losing electrons

4) Using the greek letter designation, what would be another name for 3-

hydroxybutyrate?

a) Hydroxybutyrate

b) "-hydroxybutyrate

c) !-hydroxybutyrate

d) #-hydroxybutyrate

e) $-hydroxybutyrate

5) The carbonyl group on fructose (ketohexose) is ____ and the carbonyl group on

glucose (aldohexose) is ____.

a) -C=O; -C=OH

b) -C=OH; -C=O

c) -C=O; -C=O

d) -C=OH; -C=OH

e) -COOH; -COOH

6) Which of the following statements is NOT true?

a) D-Glucose and D-Mannose are epimers

b) D-Mannose and D-Galactose are epimers

c) D-Galactose and D-Glucose are epimers

d) Enantiomers are nonsuperimposable mirror images

e) Isomers have the same chemical formula but a different arrangement of atoms

7) In their ring structures, a pyranose will have oxygen between carbons ____ and a

furanose will have oxygen between carbons ____.

a) 1 and 5; 1 and 6

b) 4 and 5; 5 and 6

c) 1 and 2; 1 and 2

d) 5 and 6; 4 and 5

e) 1 and 6; 1 and 5

8) Mutarotation of "-D-Glucopyranose would yield:

a) "-L-Glucopyranose

b) "-D-Glucofuranose

c) "-L- Glucofuranose

d) !-D-Glucopyranose

e) !-L-Glucopyranose

9) How would a sugar be named if the terminal hydroxyl group is oxidized?

a) -onic acid

b) -oate

c) -uronic acid

d) -polyol

Page 13: Quiz Biochemistry

Biochemistry 14Mar2009

DO NOT DISTRIBUTE - 13 -

e) -hydroxyl

10) Which of the following types of bonds does starch (glycogen) contain?

a) N-glycosidic

b) "-1,4, O-glycosidic

c) "-1,6, O-glycosidic

d) !, O-glycosidic

e) "-1,4, O-glycosidic and "-1,6, O-glycosidic

11) How would a fatty acid be named if it was 18 carbons long and had double bonds at

C9, C12, and C15 from the -COOH end?

a) 18:3

b) 18:3#9,12,15(%4)

c) 18:3#9,12,15(%3)

d) 18:3#9,12,15(%2)

e) 18:3#3,6,9(%9)

12) What type of double bonds cause a fatty acid chain to bend?

a) C-C

b) D

c) L

d) Cis

e) Trans

13) Free radicals, which are harmful to the body, can form from natural events (aging).

Which of the following is another way free radicals could form?

a) Chemotherapy

b) Radiation therapy

c) Bone fracture

d) Hyperkalemia

e) Hypoxia

6 – Amino Acids in Proteins

1.1) At a physiologic pH of 7.4, the amino acid amino group carries a ____ charge and

the carboxylic acid group carries a ____ charge. (hydrogen bonding can take place)

a) Neutral; Neutral

b) Positive; Negative

c) Negative; Positive

d) Positive; Positive

e) Negative; Negative

1.2) In amino acids, the pKa for the carboxylic acid group is about ____ and the pKa for

the amino group is about ____.

a) 2; 2

b) 9.5; 9.5

c) 2; 9.5

d) 9.5; 2

e) 7.4; 7.4

1.3) What type of bond holds each amino acid together in a long chain?

a) Disulfide

b) Electrostatic

Page 14: Quiz Biochemistry

Biochemistry 14Mar2009

DO NOT DISTRIBUTE - 14 -

c) Hydrogen

d) Van der Waals

e) Peptide

2.1) What is the one-letter code for asparagine?

a) A

b) S

c) G

d) N

e) E

2.2) What is the one-letter code for aspartate?

a) D

b) A

c) S

d) P

e) T

2.3) What is the one-letter code for glutamine?

a) Q

b) G

c) H

d) M

e) I

2.4) What is the one-letter code for glutamate?

a) G

b) L

c) U

d) T

e) E

3.1) Which of the following is NOT a polar, uncharged amino acid?

a) Asparagine

b) Aspartate

c) Glutamine

d) Serine

e) Threonine

3.2) Which of the following is NOT a charged amino acid?

a) Histidine

b) Lysine

c) Arginine

d) Glutamate

e) Valine

3.3) Which of the following is NOT a nonpolar, aliphatic amino acid?

a) Glycine

b) Alanine

c) Cysteine

d) Leucine

e) Isoleucine

4.1) Which of the following amino acids contains sulfur?

Page 15: Quiz Biochemistry

Biochemistry 14Mar2009

DO NOT DISTRIBUTE - 15 -

a) Methionine

b) Histidine

c) Leucine

d) Tyrosine

e) Glutamate

4.2) Which of the following amino acids is negatively charged (acidic)?

a) Lysine

b) Glutamate

c) Arginine

d) Histidine

e) Serine

5.1) What amino acid has ornithine (role in urea production) as a derivative?

a) Histidine

b) Arginine

c) Cysteine

d) Glycine

e) Serine

5.2) What amino acid has melatonin (pineal gland secretion) as a derivative?

a) Phenylalanine

b) Arginine

c) Cysteine

d) Tryptophan

e) Serine

5.3) What amino acid has histamine (role in anti-inflammatory response) as a derivative?

a) Histidine

b) Arginine

c) Cysteine

d) Glycine

e) Serine

5.4) An energy drink is found to contain taurine, a component of bile salts. What amino

acid is taurine a derivative?

a) Histidine

b) Arginine

c) Cysteine

d) Glycine

e) Serine

5.5) Which of the following amino acids is NOT a derivative for acetyl choline,

norepinephrine, or dopamine?

a) Serine

b) Threonine

c) Tyrosine

d) Methionine

e) Phenylalanine

5.6) Which of the following is made from glutamic acid?

a) Porphyrins

b) Histidine

Page 16: Quiz Biochemistry

Biochemistry 14Mar2009

DO NOT DISTRIBUTE - 16 -

c) Adrenaline

d) Thyroxin

e) #-aminobutyric acid (GABA)

6) Insulin has ____ variations (polymorphisms) in amino acid regions that affect activity.

This allows for the use of bovine and pork insulin to treat diabetes mellitus.

a) Many (poly)

b) Some (oligo)

c) Three (tri)

d) One (mono)

e) No

7) What is the term for genes that are similar to each other because they originated from a

common ancestor?

a) Homolog

b) Ortholog

c) Polymorph

d) Paralog

e) Ohnolog

8) There are at least nine different isoforms of adenylyl cyclase (synthesizes cAMP).

These isozymes allow for:

a) Quicker dampening of cell response

b) Cells to response similarly to the same hormone

c) Cells to response similarly to a different hormone

d) Cells to respond differently to the same hormone

e) Cells to respond differently to a different hormone

9) Isozymes are enzymes with different amino acid sequences but the same function.

Creatine kinase is created in two distinct forms in the body. Which of the following has a

mix of these forms, allowing for tissue recognition during damage?

a) Brain

b) Skeletal muscle

c) Cardiac muscle

d) Smooth muscle

e) Pancreas

10) Which of the following is NOT true regarding insulin and C-peptide?

a) C-peptide functions in repair of the muscular layer of the arteries

b) C-peptide connects the A and B chains of insulin

c) Proinsulin splits into insulin and C-peptide

d) Proinsulin is cleaved in three locations

e) Bovine and pork insulin would cause an immune response in all human patients

11) Which of the following is NOT a post-translational modification of amino acids?

a) Glycosylation

b) Isozymation

c) Fatty acylation

d) Prenylation

e) Regulatory modification

7 – Structure/Function Relationships in Proteins

Page 17: Quiz Biochemistry

Biochemistry 14Mar2009

DO NOT DISTRIBUTE - 17 -

1) How are amino acids arranged in a primary protein structure?

a) Helical

b) Crossed

c) Folded

d) Perpendicular

e) Linear

2.1) Which of the following characterizes "-helix regions of proteins?

a) They all have the same primary structure

b) They are formed principally by hydrogen bonds between a carbonyl oxygen

atom in one peptide bond and the amide hydrogen from a different peptide bond

c) They are formed principally by hydrogen bonds between a carbonyl atom in

one peptide bond and the hydrogen atoms on the side chain of another amino acid

d) They are formed by hydrogen bonding between two adjacent amino acids in

the primary sequence

e) They require a high content of proline and glycine

2.2) In contrast with the "-helix, !-sheets form:

a) With the peptide hydrogen bonds on the same strand

b) With the peptide hydrogen bonds on an adjacent strand

c) With covalent bonding

d) With ionic bonding

e) With peptide bonds between a carbonyl oxygen and an amide

3.1) What helps distinguish globular proteins from fibrous proteins?

a) "-helices

b) !-sheets

c) Molecular weight

d) Freezing/melting point

e) Solubility

3.2) In the actin fold of globular proteins, ____ is bound into the middle of the cleft by

amino acid residues, which promotes a conformational change and cleaves to ____.

a) cAMP; AMP

b) cAMP; ADP + P

c) ATP; AMP

d) ATP; ADP + P

e) Actin; Myosin

4) What type of proteins are geometrically linear, arranged around a single axis, and have

a repeating unit structure?

a) Globular

b) Fibrous

c) Membrane-spanning

5) What protein structure refers to the association of individual polypeptide chain

subunits in a geometrically and stoichiometrically specific manner?

a) Primary

b) "-helix

c) !-sheet

d) Tertiary

e) Quaternary

Page 18: Quiz Biochemistry

Biochemistry 14Mar2009

DO NOT DISTRIBUTE - 18 -

6.1) Which of the following is characteristic of globular proteins?

a) Hydrophilic amino acids tend to be on the inside

b) Hydrophobic amino acids tend to be on the outside

c) Tertiary structure is formed by hydrophobic and electrostatic interactions

between amino acids, and by hydrogen bonds between amino acids and between

amino acids and water

d) Secondary structures are formed principally by hydrophobic interactions

between amino acids

e) Covalent disulfide bonds are necessary to hold the protein in a rigid

conformation

6.2) A protein has one transmembrane domain composed entirely of an "-helical

secondary structure. Which of the following amino acids would you expect to find in the

transmembrane domain?

a) Proline

b) Glutamate

c) Lysine

d) Leucine

e) Arginine

6.3) Autopsies of patients with Alzheimer disease show protein aggregates called

neurofibrillary tangles and neuritic plaques in various regions of the brain. These plaques

exhibit the characteristic staining of amyloid. Which of the following structural features

is the most likely characteristic of at least one protein in these plaques?

a) A high content of !-pleated sheet structure

b) A high content of "-helical structure

c) A high content of random coils

d) Disulfide bond crosslinks between polypeptide chains

e) A low-energy native conformation

7) At lower levels of pO2 in oxygen-using tissues, hemoglobin cannot bind oxygen as

well as myoglobin. This is because myoglobin is a globular protein composed of a single

polypeptide chain with ____ O2 binding site(s), and hemoglobin has ____ binding site(s).

a) 2; 2

b) 1; 1

c) 1; 2

d) 1; 3

e) 1; 4

8.1) When the amount of oxygen bound to myoglobin or hemoglobin is plotted against

the partial pressure of oxygen (pO2), myoglobin has a ____ curve and hemoglobin has a

____ curve.

a) Sigmoidal; Hyperbolic

b) Hyperbolic, Sigmoidal

c) Linear; Hyperbolic

d) Sigmoidal; Linear

e) Linear; Sigmoidal

8.2) In the hemoglobin T (tense) state there is ____ affinity for O2 and in the R (relaxed)

state there is ____ affinity for O2.

a) Low; No

Page 19: Quiz Biochemistry

Biochemistry 14Mar2009

DO NOT DISTRIBUTE - 19 -

b) High; Low

c) Low; High

d) High; No

e) No; No

9) Each immunoglobulin (IgG) molecule contains two light (L) and two heavy (H) chains

joined by disulfide bonds. Each light chain has a variable domain (VL) and a region of

constant amino acid sequence (CL). Each heavy chain has ____ variable domain(s) (VH)

and ____ constant domain(s) (CH).

a) 2; 2

b) 1; 1

c) 1; 2

d) 1; 3

e) 1; 4

10) What determines how a proteins folds?

a) Primary structure

b) "-helix structure

c) !-sheet structure

d) Tertiary structure

e) Quaternary structure

11) Which of the following would NOT cause denaturization of a protein (loss of

function)?

a) Nonenzymatic glycosylation

b) Temperature changes

c) pH changes

d) Solvent used

e) !-pleated sheet formation

12) In sickle-cell disease, single point valine substitution on the beta chain of hemoglobin

creates a knob, which can bind into a groove on the alpha chain, with oxygen saturation

dependent affinity. At high oxygen saturations (lungs), the chains ____. At low oxygen

saturations (capillaries), the chains ____.

a) Do not bind together; Bind together to form polymers

b) Bind together to form polymers; Do not bind together

c) Break down; Bind together to form polymers

d) Bind together to form polymers; Break down

e) Break down; Break down

13) Measurement of which of the following can aid diagnosis of crush injury necrosis and

myocardial infarction soon after an event?

a) Hemoglobin

b) Myoglobin

c) Troponin

d) Actin

e) Myosin

14.1) Which of the following prion diseases is associated with ritualistic cannibalism in

the Fore tribes people?

a) Mad-cow disease (bovine spongiform encephalopathy)

b) Creutzfeldt-Jakob disease (CJD)

Page 20: Quiz Biochemistry

Biochemistry 14Mar2009

DO NOT DISTRIBUTE - 20 -

c) Iatrogenic CJD

d) Kuru

e) Scrapie

14.2) Which of the following prion diseases is associated with infection?

a) Mad-cow disease (bovine spongiform encephalopathy)

b) Creutzfeldt-Jakob disease (CJD)

c) Iatrogenic CJD

d) Kuru

e) Gerstmann-Straussler-Scheinker Syndrome

8 – Enzymes as Catalysts

1) Which of the following is NOT true about enzymes?

a) They catalyze reactions

b) The provide specificity to reactions

c) They bind to the reaction product

d) They provide regulatory control to reactions

e) They are usually proteins

2) Which of the following involves a conformational change to a substrate active site

with binding of an enzyme?

a) Lock-and-key

b) Induced fit

c) Coenzymes

d) Competitive inhibition

e) Competitive catalyst

3.1) Most coenzymes are derived from:

a) Carbohydrates

b) Triacylglycerols

c) Proteins

d) Vitamins

e) Minerals

3.2) In the absence of their enzyme, coenzymes have ____ activity and ____ specificity.

a) Weak; Strong

b) Strong; Weak

c) Weak; Weak

d) Strong; Strong

3.3) Activation-transfer coenzymes usually participate directly in catalysis by forming

what kind of bonds?

a) Hydrogen

b) Van der Waals

c) Covalent

d) Ionic

e) Electron transfer

3.4) Oxidation-reduction coenzymes form what kind of bonds?

a) Hydrogen

b) Van der Waals

c) Covalent

Page 21: Quiz Biochemistry

Biochemistry 14Mar2009

DO NOT DISTRIBUTE - 21 -

d) Disulfide

e) Electron transfer

3.5) What cofactor (metal ion) is needed for the binding of ATP?

a) Mg++

b) Ca++

c) K+

d) Na+

e) Cl-

3.6) A patient was born with a congenital mutation in an enzyme, severely affecting its

ability to bind an activation-transfer coenzyme. As a consequence:

a) The enzyme would be unable to bind the substrate of the reaction

b) The enzyme would be unable to form the transition state complex

c) The enzyme would normally use a different activation-transfer coenzyme

d) The enzyme would normally substitute the functional group of an active site

amino acid residue for the coenzyme

e) The reaction could be carried out by the free coenzyme, provided the diet

carried an adequate amount of its vitamin precursor

4.1) In general, enzymes are the most effective when pH is ____ and temperature is about

____ degrees Celsius.

a) Neutral; 37

b) Basic; 37

c) Acidic; 37

d) Neutral; 25

e) Basic; 25

4.2) A patient developed a bacterial overgrowth in his intestine that decreased the pH of

the luminal contents from their normal pH of approximately 6.5 down to 5.5. This

decrease in pH is likely to:

a) Denature proteins reaching the intestine with their native structure intact

b) Disrupt hydrogen bonding essential for maintenance of tertiary structure

c) Inhibit intestinal enzymes dependent on histidine for acid-base catalysis

d) Inhibit intestinal enzymes dependent on active site lysine for binding substrate

e) Have little effect on hydrolases

5.1) Which of the following is NOT true regarding enzymes?

a) Enzymes are proteins and can be denatured

b) Enzymes act at a very low concentration

c) Enzymes will only react with one or a very small number of compounds

d) Enzymes are very small molecules, much smaller than their substrates

e) Some enzymes require the presence of one or more metal ions for activity

Match the description (enzyme inhibition mechanism) with the drug:

A) Oganophosphate, B) Aspirin, C) Penicillin, D) Allopurinol

5.2) Transition state analog that binds to glycopeptidyl transferase

5.3) Inhibition of acetylcholinesterase

5.4) Suicide inhibitor of xanthine oxidase. Used as a treatment for gout.

5.5) Covalent acetylation of serine in the enzyme prostaglandin endoperoxide synthase

Match the description with the therapeutic agent:

6.1) Activates the fibrinolytic enzyme system a) Asparaginase

Page 22: Quiz Biochemistry

Biochemistry 14Mar2009

DO NOT DISTRIBUTE - 22 -

6.2) Produced form E. coli to dissolve MI clots b) Tissue plasminogen activator

6.3) Used in treatment of certain types of leukemia c) Streptokinase

7) Which of the following enzymes in serum following an uncomplicated myocardial

infarction (MI) would be the first to increase above its upper limit?

a) ALT - Alamine aminotransferase

b) AST - Aspartate

c) CK - Creatine Kinase

d) LDH - Lactate dehydrogenase

9 – Regulations of Enzymes

1.1) Which of the following describes a characteristic feature of an enzyme obeying

Michaelis-Menten kinetics?

a) The enzyme velocity is at 1/2 the maximal rate when 100% of the enzyme

molecules contain bound substrate

b) The enzyme velocity is at 1/2 the maximal rate when 50% of the enzyme

molecules contain bound substrate

c) The enzyme velocity is at its maximal rate when 50% of the enzyme molecules

contain bound substrate

d) The enzyme velocity is at its maximal rate when all of the substrate molecules

in solution are bound by the enzyme

e) The velocity of the reaction is independent of the concentration of enzyme

1.2) If Km for glucose is 5mM, what is the initial and final velocity with regard to Vmax

(using Michaelis-Menten equation) if blood glucose goes from 5mM to 20mM?

a) 0.50 Vmax; 0.80 Vmax

b) 1.0 Vmax; 0.75 Vmax

c) 0.25 Vmax; 0.125 Vmax

d) 0.25 Vmax; 0.40 Vmax

e) 0.50 Vmax; 0.75 Vmax

1.3) Enzyme ____ is dependent on (proportional to) substrate concentration and enzyme

____ is dependent on (proportional to) enzyme concentration.

a) Reactant; Product

b) Product; Reactant

c) Velocity; Product

d) Product; Velocity

e) Velocity; Reactant

2) Km of the enzyme for a substrate is the concentration of substrate required to reach:

a) Vmax

b) 3/4 Vmax

c) 1/2 Vmax

d) 1/4 Vmax

e) Km and Vmax are independent of each other

3.1) In ____ inhibition, Km increases and Vmax does not change. In ____ inhibition,

Vmax changes and Km does not.

a) Non-competitive; Un-competitive

b) Un-competitive; Competitive

c) Competitive; Un-competitive

Page 23: Quiz Biochemistry

Biochemistry 14Mar2009

DO NOT DISTRIBUTE - 23 -

d) Non-competitive; Competitive

e) Competitive; Non-competitive

3.2) What type of inhibition is mixed in that it changes both Vmax and Km?

a) Competitive

b) Non-competitive

c) Un-competitive

4.1) Which of the following describes a characteristic of most allosteric enzymes?

a) They are composed of single subunits

b) In the absence of effectors, they generally follow Michaelis-Menten kinetics

c) They show cooperativity in substrate binding

d) They have allosteric activators that bind in the catalytic site

e) They have irreversible allosteric inhibitors that bind at allosteric sites

4.2) In generally, activators of allosteric enzymes bind more tightly to the high-affinity T

state of the enzyme than the T state.

a) True

b) False

5) In a plot of velocity (vi/Vmax) versus substrate concentration, an activator would shift

the curve to the ____, change ____, but not change ____.

a) Left; Km; [S]

b) Right; Vmax; Km

c) Left; Vmax; Km

d) Right; Km; Vmax

e) Left; Km; Vmax

6.1) What is the rate-limiting enzyme in the pathway of glycogen degradation, degrading

to glucose 1-phosphate and regulated by AMP?

a) Glycolase

b) Amylase

c) Glycogen phosphorylase

d) Protein kinase

e) Glyceraldehyde

6.2) Adrenaline indirectly increases ____, which activates protein kinase A, starting the

phosphorylation cascade.

a) Phosphorylase kinase

b) Glycogen phosphorylase a

c) Glycogen phosphorylase b

d) cAMP

e) cGMP

7) Ca++/calmodulin is an example of a dissociable modulator protein that binds to a

number of different proteins and regulates their function. This is done by:

a) Modifying the enzyme

b) Inhibition of the catalyst

c) Addition of a coenzyme

d) Modifying the catalytic site

e) Blocking glycogen phosphorylase kinase

8) Inactive precursor enzymes (zymogens) must undergo ____ to be activated.

a) Reversible proteolytic cleavage

Page 24: Quiz Biochemistry

Biochemistry 14Mar2009

DO NOT DISTRIBUTE - 24 -

b) Irreversible proteolytic cleavage

c) Lysosomal activation

d) Secretory vesicle secretion

e) Cleavage during synthesis

9) In a Lineweaver-Burk plot, a competitive inhibitor will have a(n) ____ slope with the

same y-axis (1/Vmax) as no inhibitor, and a pure non-competitive inhibitor will have a(n)

____ slope with the same x-axis (-1/Km) as no inhibitor.

a) Increased; Decreased

b) Decreased; Increased

c) Increased; Increased

d) Decreased; Decreased

10 – Relationship between Cell Biology and Biochemistry

1.1) Which of the following is NOT a major lipid component of plasma membranes?

a) Sphingomyelin

b) Phosphatidyl-choline

c) Ethanolamine

d) Diphosphatidyl glycerol

e) Phosphatidyl-serine

1.2) Which of the following is a characteristic of the plasma membrane?

a) It is composed principally of triacylglycerols and cholesterol

b) It contains principally nonpolar lipids

c) It contains phospholipids and their acyl groups extending into the cytosol

d) It contains more phosphatidylserine in the inner than outer leaflet

e) It contains oligosaccharides sandwiched between the inner and outer leaflets

2) Cholesterol maintains membrane fluidity by binding to the ____ pocket of

phophoacylglycerols, binding with cholesterol’s ____ group.

a) Cis; Hydroxyl

b) Trans; Hydroxyl

c) Cis; Hydrogen

d) Trans; Hydrogen

3) Which of the following transport mechanisms uses ATP but does not directly couple

with the ATP, and the electrochemical potential difference created by pumping ions out

of the cell is used?

a) Simple diffusion

b) Facilitated diffusion

c) Ligand-gated channels

d) Active transport

e) Secondary active transport

4.1) In the active transport Na+/K+ ATPase pump, ____ Na+ ion(s) is/are pumped out of

the cell and ____ K+ ion(s) is/are pumped into the cell.

a) 1; 1

b) 2; 2

c) 3; 3

d) 3; 2

e) 3; 1

Page 25: Quiz Biochemistry

Biochemistry 14Mar2009

DO NOT DISTRIBUTE - 25 -

4.2) The Na+/glucose cotransporter uses a(n) ____ transport mechanism and involves

pumping in ____ Na+ ion(s) with glucose into the cell.

a) Active; One

b) Secondary active; One

c) Ligand-gated; One

d) Active; Two

e) Secondary active; Two

5) Most lysosomal enzymes are:

a) Proteases

b) Glycosidases

c) Hydrolases

d) Sulfatases

e) Nucleases

f) Lipases

6) The ____ process involves clatherin-coated pits. The ____ process involves AA, FA,

and CHOs being released into the cytosol and undigested material forming lipofuscin

granules.

a) Receptor-mediated endocytosis; Phagocytosis

b) Receptor-mediated endocytosis; Receptor-mediated endocytosis

c) Phagocytosis; Receptor-mediated endocytosis

d) Phagocytosis; Phagocytosis

7.1) The electron transport chain (ETC) takes place in the ____ membrane of the

mitochondria and the tricarboxylic acid cycle (TCA) takes place in the ____ membrane

of the mitochondria.

a) Outer; Inner

b) Inner; Outer

c) Outer; Outer

d) Inner; Inner

e) Neither take place within the mitochondria

7.2) The cristae are infoldings in the mitochondria to make more space for:

a) TCA cycle

b) Cory cycle

c) Glycolysis

d) Gluconeogenesis

e) ETC

11 – Cell Signaling by Chemical Messengers

1.1) Which of the following is a general characteristic of all chemical messengers?

a) They are secreted by one cell, enter the blood, and act on a distant target cell

b) To achieve a coordinated response, each messenger is secreted by several types

of cells

c) Each messenger binds to a specific protein receptor in a target cell

d) Chemical messengers must enter cells to transmit their message

e) Chemical messengers are metabolized to intracellular second messengers to

transmit their message

Page 26: Quiz Biochemistry

Biochemistry 14Mar2009

DO NOT DISTRIBUTE - 26 -

1.2) Which of the following is a characteristic of chemical messengers that bind to

intracellular transcription factor receptors?

a) They are usually cytokines or polypeptide hormones

b) They are usually small molecule neurotransmitters

c) They exert rapid actions in cells

d) They are transported through the blood bound to proteins

e) They are always present in high concentrations in the blood

2) During neurotransmission in a muscle cell, what ion triggers fusion of the vesicle with

the plasma membrane, causing acetylcholine to be released into the synaptic cleft?

a) Na+

b) Ca++

c) Cl-

d) K+

e) Mg++

3) The nicotinic acetylcholine receptor, which allows Na+ into the cell and K+ out of the

cell, is made up of ____ subunits with ____ membrane-spanning helical regions per

subunit.

a) 4; 4

b) 5; 5

c) 4; 5

d) 5; 4

4) The drug edrophonium chloride is given for temporary treatment of myasthenia gravis

(MG). This drug works by:

a) Inhibits Ca++ release

b) Inhibits T-lymphocytes

c) Inhibits B-lymphocytes

d) Inhibits acetylcholine

e) Inhibits acetylcholinesterase

5) Which type of chemical messengers are important in limiting the immune response to

a specific location in the body, a feature that helps prevent the development of

autoimmune disease?

a) Endocrine

b) Paracrine

c) Autocrine

6.1) What is the function of cytokines?

a) Transmit signals at synaptic junctions

b) Regulate metabolism via the blood stream

c) Kill invading microorganisms

d) Respond to cell injury

e) Stimulate cell proliferation

6.2) What is the function of eicosanoids (e.g. prostaglandins, thromboxanes,

leukotrienes)?

a) Transmit signals at synaptic junctions

b) Regulate metabolism via the blood stream

c) Kill invading microorganisms

d) Respond to cell injury

Page 27: Quiz Biochemistry

Biochemistry 14Mar2009

DO NOT DISTRIBUTE - 27 -

e) Stimulate cell proliferation

7) Which of the following would most likely bind to an intracellular receptor versus a

plasma membrane receptor?

a) Steroid hormones

b) Peptide hormones

c) Cytokines

d) Catecholamines

8) Heptahelical receptors, which contain 7-membrane spanning "-helices, are the most

common type of plasma membrane receptors. They work through:

a) Simple diffusion

b) Facilitated diffusion

c) Second messengers

d) Active transport

e) Secondary active transport

9) Tyrosine kinase receptors include Ras, MAP, phosphatidylinositol phosphates, and

insulin. Once a receptor ____ is formed, tyrosine kinase domains phosphorylate each

other and the residues form binding sites for signal transducer proteins.

a) Monomer

b) Dimer

c) Trimer

d) Oligomer

e) Polymer

10.1) JAK-STAT receptors, which have no intrinsic kinase activity, are often used to:

a) Transmit signals at synaptic junctions

b) Regulate metabolism via the blood stream

c) Kill invading microorganisms

d) Respond to cell injury

e) Stimulate cell proliferation

10.2) What type of receptors are associated with the Smad family of proteins?

a) JAK-STAT

b) PIP2

c) Insulin receptors

d) Serine/Threonine kinases

e) Ras and MAP kinases

11.1) Following the activation of heptahelical receptors, which subunit of the

heterotrimeric G protein is released as GTP is exchanged in for GDP?

a) " (alpha)

b) ! (beta)

c) # (gamma)

11.2) What is the cellular target/goal of diacrylglycerol (DAG)?

a) PKA

b) PKG

c) PKC

d) Calmodulin

e) Ca++ release from ER/SR

11.3) What is the cellular target/goal of inositol triphosphate (IP3)?

Page 28: Quiz Biochemistry

Biochemistry 14Mar2009

DO NOT DISTRIBUTE - 28 -

a) PKA

b) PKG

c) PKC

d) Calmodulin

e) Ca++ release from ER/SR

12) What secondary messenger(s) is/are created when phospholipase C (PLC) catalyzes

the hydrolysis of phosphatidylinositol?

a) cAMP

b) cGMP and cAMP

c) IP3

d) DAG

e) DAG and IP3

13) Which of the following is NOT a signaling pathway termination method?

a) Stimulus is taken away

b) Stimulus is increased to over-saturate the receptor

c) Signal is degraded by an esterase

d) Second messenger is degraded

e) Receptor is phosphorylated or dephosphorylated

14) Nitric oxide (NO) is an exception to the rule that intracellular receptors are gene

transcription factors since it is a lipophilic gas that can diffuse into the cell. NO is a

receptor associated with:

a) cAMP

b) cGMP

c) Ca++

d) DAG

e) IP3

12 – Structure of the Nucleic Acids

1.1) Euchromatin is diffuse chromatin seen during what cell phase?

a) Prophase

b) Metaphase

c) Anaphase

d) Telophase

e) Interphase

1.2) The poly (A) tail is a sequence of ____ nucleotides at the ____ end.

a) Adenine; 3’

b) Guanine; 3’

c) Adenine; 5’

d) Guanine; 5’

1.3) In deoxyribose (sugar in DNA), what ribose carbon lacks a hydroxyl group?

a) C1

b) C2

c) C3

d) C4

e) C5

Page 29: Quiz Biochemistry

Biochemistry 14Mar2009

DO NOT DISTRIBUTE - 29 -

2.1) A polynucleotide chain of DNA is held together by what molecule between the 3’

and 5’ carbons of adjacent nucleotides?

a) Sodium

b) Magnesium

c) Phosphate

d) Sulfide

e) Oxygen

2.2) In DNA, the bond between the deoxyribose sugar and the molecule above question is

which of the following?

a) Polar

b) Ionic

c) Hydrogen

d) Covalent

e) Van der Waals

3) AT-rich DNA would denature before GC-rich DNA because adenine and thymine are

stabilized by ____ hydrogen bonds, while guanine and cytosine are stabilized by ____

hydrogen bonds.

a) 1; 2

b) 2; 1

c) 2; 3

d) 3; 2

e) 1; 3

4.1) For the following DNA sequence, determine the sequence and direction of the

complementary strand: 5’-ATCGATCGATCGATCG-3’

a) 5’-ATCTATCGATCGATCG-3’

b) 3’-ATCGATCGATCGATCG-5’

c) 5’-CGAUCGAUCAUCGAU-3’

d) 5’-CGATCGATCGATCGAT-3’

e) 3’-CGATCGATCGATCGAT-3’

4.2) DNA exhibits directionality in what form?

a) 3’ => 3’

b) 5’ => 3’

c) 3’ => 5’

d) 5’ => 5’

5) Prokaryote ribosomes are 70S formed by ____ and eukaryote ribosomes are 80S

formed by ____.

a) 60S and 20S; 50S and 40S

b) 60S and 50S; 40S and 30S

c) 50S and 20S; 60S and 20S

d) 60S and 40S; 50S and 30S

e) 50S and 30S; 60S and 40S

6) Which of the following is NOT a core histone, involved in the packing of DNA?

a) H4

b) H3

c) H1

d) H2A

Page 30: Quiz Biochemistry

Biochemistry 14Mar2009

DO NOT DISTRIBUTE - 30 -

e) H2B

7.1) What type of RNA carries amino acids to ribosomes and ensures that they are

incorporated into the appropriate positions in the growing polypeptide chain?

a) RNA

b) mRNA

c) rRNA

d) tRNA

e) snRNP

7.2) What type of RNA is converted into the amino acid sequence of a polypeptide chain?

a) RNA

b) mRNA

c) rRNA

d) tRNA

e) snRNP

13 – Synthesis of DNA

1) What is the function of ligase?

a) Works with continuous 5’ => 3’ synthesis

b) Works on the leading strand

c) Undoes the work of DNA polymerase

d) Joins the Okazaki fragments

e) Winds the leading and lagging strands together

2) Which of the following relieves the torsional strain on the parental duplex of DNA

caused by unwinding during synthesis?

a) DNA polymerase

b) Primase

c) Helicase

d) DNA ligase

e) Topoisomerase

3.1) The key mechanistic failure in patient with xeroderma pigmentosum involves which

of the following?

a) Mutation in the primase gene

b) Inability to excise a section of the UV-damaged DNA

c) Mutation of one of the mismatched repair components

d) Inability to synthesize DNA across the damaged region

e) Loss of proofreading capacity

3.2) What can cause two adjacent pyrimidines to form a covalent dimmer (DNA thymine

dimmer)?

a) UV light

b) High pH

c) Low pH

d) Benzo[a]pyrene

e) High heat

3.3) What type of DNA repair acts when replication finds an incorrect, but normal base?

a) Nucleotide excision repair

b) Base excision repair

Page 31: Quiz Biochemistry

Biochemistry 14Mar2009

DO NOT DISTRIBUTE - 31 -

c) Mismatch repair

d) Transcription-coupled repair

4) Benzo[a]pyrene, from cigarette smoke, becomes carcinogenic when it is:

a) Hydrolyzed

b) Reduced

c) Oxidized

d) Phosphorylated

e) Cleaved

5) ddIs terminate cell grown (DNA antiviral medication) when converted to ddCs

because of the absence of a ____ group on either the 2’ or 3’ carbon.

a) Hydrogen

b) Phosphate

c) Ester

d) Hydroxyl

e) Amino

14 – Transcription: Synthesis of RNA

1.1) Which of the following is considered the promoter sequence (Pribnow box) in the

prokaryote E. coli?

a) TATAAT

b) PyAPy

c) AUG

d) ATG

e) AGGT

1.2) Heterogeneous nuclear RNA (hnRNA) contains both intronic and exonic sequences.

It is modified such that a ____ is added to the 5’ end and a ____ is added to the 3’ end.

a) Promoter; Cap

b) Cap; Poly (A) tail

c) Poly (A) tail; Cap

d) Cap; Promoter

e) Promoter; Terminator

2.1) The short transcript AUCCGUACG would be derived from which of the following

DNA sequences? (all are 5’ to 3’)

a) ATCCGTACG

b) CGTACGGAT

c) AUCCGUACG

d) TAGGCATGC

e) GCATGCCTA

2.2) In a segment of a transcribed gene, the nontemplate strand of DNA has the following

sequence: 5’-AGCTCACTG-3’ What will be the corresponding sequence in the RNA

produced from this segment of the gene?

a) CAGUGAGCU

b) AGCUCACUG

c) CAGTGAGCT

d) UCGAGUGAC

e) GTCACTCGA

Page 32: Quiz Biochemistry

Biochemistry 14Mar2009

DO NOT DISTRIBUTE - 32 -

2.3) Which of the following eukaryotic DNA control sequences does not need to be in a

fixed location, and is most responsible for high rates of transcription of particular genes?

a) Promoter

b) Promoter-proximal element

c) Enhancer

d) Operator

e) Splice donor site

2.4) Which of the following is true of both eukaryotic and prokaryotic gene expression?

a) After transcription, a 3’ poly A tail and a 5’ cap are added to mRNA

b) Translation of mRNA can begin before transcription is complete

c) mRNA is synthesized in the 3’ o 5’ direction

d) RNA polymerase binds at a promoter region upstream to the gene

e) Mature mRNA is always precisely co-linear to the gene from which it was

transcribed

2.5) DNA is read in the ____ direction and RNA is synthesized in the ____ direction.

a) 5’ => 3’; 5’ => 3’

b) 5’ => 3’; 3’ => 5’

c) 3’ => 5’; 5’ => 3’

d) 3’ => 5’; 3’ => 5’

2.6) The coding strand of DNA is the same as the associated mRNA EXCEPT for:

a) mRNA uses U instead of T

b) mRNA uses T instead of U

c) mRNA uses G instead of C

d) mRNA uses C instead of G

e) mRNA uses A instead of T

3) Which of the following is the correct sequence leading to mRNA synthesis?

a) hnRNA => Capping => Poly A tail => Exon removal

b) pre-RNA => Poly A tail => Capping => Intron removal

c) pre-RNA => Capping => Poly A tail => Intron removal

d) hnRNA => Poly A tail => Capping => Intron removal

e) hnRNA => Capping => Poly A tail => Intron removal

4.1) Enhancer elements ____ affect the ____ of transcription.

a) Negatively; Speed

b) Negatively; Frequency

c) Positively; Speed

d) Positively; Frequency

4.2) The proximal element could be found immediately ____ to the ____.

a) 3’; Cap

b) 3’; Poly A tail

c) 5’; Cap

d) 5’; Poly A tail

4.3) What elements are proteins that bind to DNA and facilitate or prevent the binding of

RNA polymerase?

a) Cis-actin

b) Trans-actin

c) Distal

Page 33: Quiz Biochemistry

Biochemistry 14Mar2009

DO NOT DISTRIBUTE - 33 -

d) Proximal

e) Silencer

5.1) Rifampin/Rifamycin is a tuberculosis drug that inhibits ____ in ____.

a) DNA polymerase; Eukaryotes

b) DNA polymerase; Prokaryotes

c) RNA polymerase; Eukaryotes

d) RNA polymerase; Prokaryotes

5.2) Zidovudine (AZT), a drug given to treat HIV, inhibits viral:

a) RNA polymerase

b) DNA polymerase

c) Reverse transcriptase

d) DNA ligase

e) Topoisomerase

5.3) "-amanitin, a poison found in certain mushrooms, inhibits ____ in ____.

a) DNA polymerase; Eukaryotes

b) DNA polymerase; Prokaryotes

c) RNA polymerase; Eukaryotes

d) RNA polymerase; Prokaryotes

15 – Translation: Synthesis of Proteins

1) DNA is synthesized and transcribed to mRNA, which is translated (tRNA on

ribosomes) to a protein. This process can be circumvented by which of the following

enzymes?

a) RNA polymerase

b) DNA polymerase

c) Reverse transcriptase

d) DNA ligase

e) Topoisomerase

2) The anticodon is a set of three nucleotides on the 3’-end of ____, which corresponds to

the codon on the ____.

a) tRNA; mRNA

b) mRNA; tRNA

c) tRNA; rRNA

d) rRNA; tRNA

e) tRNA; hnRNA

Match the following descriptions with their terms:

3.1) All organisms use the same genetic code a) Degenerate

3.2) Only one codon specifies one amino acid b) Unambiguous

3.3) Each codon is read only once c) Non-Overlapping

3.4) An amino acid may have more than one codon d) Universality

3.5) Where is there an exception to universaility such that certain nucleotide sequences

do not code as they do elsewhere?

a) Golgi

b) Cytoplasm

c) Nucleus

d) Lysosome

Page 34: Quiz Biochemistry

Biochemistry 14Mar2009

DO NOT DISTRIBUTE - 34 -

e) Mitochondria

4.1) What type of mutations involves a change that produces a stop codon?

a) Point

b) Silent

c) Missense

d) Nonsense

e) Deletion

4.2) What type of mutation involves a change that specifies the same amino acid?

a) Point

b) Silent

c) Missense

d) Nonsense

e) Deletion

5.1) In the read-out of the genetic code in prokaryotes, which of the following processes

acts before any of the others?

a) tRNAi alignment with mRNA

b) Termination of transcription

c) Movement of the ribosome from one codon to the next

d) Recruitment of termination factors to the A site

e) Export of mRNA from the nucleus

5.2) During the initiation of translation in eukaryotes, a tRNA complex is formed with:

a) His (CAU)

b) Ser (UCG)

c) Met (AUG)

d) Gly (GGG)

e) Val (GUA)

5.3) During the elongation phase of translation in eukaryotes, the start codon is located at

the ____ site with the ____ determining what will form a peptide bond at the A site.

a) P; Codon

b) P; Anticodon

c) A; Codon

d) A; Anticodon

5.4) What causes peptidyltransferase to hydrolyze the bond between the peptide chain

and tRNA, terminating translation?

a) tRNA codon

b) tRNA anticodon

c) mRNA condon

d) tRNA stop codon

e) Release factors

6) Which of the following is NOT considered a post-translational modification?

a) Fatty acylation

b) Methylation

c) Prenylation

d) Aminoacylation

e) ADP-ribosylation

Page 35: Quiz Biochemistry

Biochemistry 14Mar2009

DO NOT DISTRIBUTE - 35 -

7) I-cell disease (Mucolipidosis II) is a disorder of protein targeting where ____ proteins

are not sorted properly from the Golgi to the target.

a) Mitochondrial

b) RER

c) SER

d) Lysosomal

e) Nucleic

Match the site/mode of action with the antibiotic:

8.1) Binds to 50S, prevents translocation a) Streptomycin

8.2) Binds to 30S, inhibits aminoacyl-tRNA b) Tetracycline

8.3) Binds to 50S, inhibits peptidyltransferase c) Chloramphenicol

8.4) Binds to 30S, prevents initiation complex d) Erythromycin

16 – Regulation of Gene Expression

1) Each eukaryotic gene has its own promoter. In prokaryotes (bacteria), genes are

organized in:

a) Specific transcription factors

b) Transactivators

c) Promoter-proximal elements

d) Response elements

e) Operons

2.1) The inducer exhibits positive control by binding to a ____ molecule, ____ the

process of transcription.

a) mRNA operator; Increasing

b) mRNA operator; Decreasing

c) Repressor molecule; Increasing

d) Repressor molecule; Decreasing

e) Co-repressor; Increasing

2.2) Catabolism of alternative sugar sources is repressed, cAMP levels are low, RNA pol

is not recruited, and the lac operon is not transcribed in the presence of:

a) Insulin

b) Glucagon

c) Glycogen

d) Glucose

e) Lactose

2.3) Attenuation of transcription of the Trp operon occurs with ____ levels of Trp,

exhibiting ____ control.

a) High; Negative

b) High; Positive

c) Low; Negative

d) Low; Positive

3.1) Which of the following describes that segments of DNA can move from one location

to another in the genome to allow the production of different proteins (e.g. production of

antibodies)?

a) Transcriptional control

b) Post-transcriptional control

Page 36: Quiz Biochemistry

Biochemistry 14Mar2009

DO NOT DISTRIBUTE - 36 -

c) Translational control

d) Post-translational control

e) Gene alteration

3.2) Which of the following would explain globin being synthesized when heme levels

are high in the cell but not when they are low? (eIFs regulated by phosphorylation)

a) Transcriptional control

b) Post-transcriptional control

c) Translational control

d) Post-translational control

e) Gene alteration

3.3) Which of the following involves regulating the assembly of the transcription

complex containing the RNA pol and its binding to the TATA box and the promoter?

a) Transcriptional control

b) Post-transcriptional control

c) Translational control

d) Post-translational control

e) Gene alteration

17 – Use of Recombinant DNA Techniques in Medicine

Will be covered in the Molecular Genetics Module

18 – The Molecular Biology of Cancer

Will be covered in the Oncology Module

19 – Cellular Bioenergentics: ATP and O2

1.1) A negative value of #G(0) of ATP hydrolysis (equimolar substrates and products)

indicates that the reaction will proceed in a ____ direction with the ____ of energy.

a) Reverse; Gain

b) Reverse; Release

c) Forward; Gain

d) Forward; Release

1.2) Which statement best describes the direction a chemical reaction will follow?

a) Under standard conditions, a reaction will proceed in the forward direction if

the free energy #G(0) is positive

b) The direction of a reaction is independent of the initial substrate and product

concentrations because the direction is determined by the change in free energy

c) The concentration of all of the substrates must be higher than all of the

products to proceed in the forward direction

d) The enzyme for the reaction must be working at better than 50% of its

maximum efficiency for the reaction to proceed in the forward direction

e) A reaction with a positive free energy will proceed in the forward direction if

the substrate concentration is raised high enough

1.3) Which of the following bioenergetic terms or phrases is correctly defined?

a) The first law of thermodynamics states that the universe tends toward a state of

increased order

Page 37: Quiz Biochemistry

Biochemistry 14Mar2009

DO NOT DISTRIBUTE - 37 -

b) The second law of thermodynamics states that the total energy of a system

remains constant

c) The change in enthalpy of a reaction is a measure of the total amount of heat

that can be released from changes in the chemical bonds

d) #G(0) of a reaction is the standard free energy measured at 37ºC and pH of 7.4

e) A high-energy bond is a bond that releases more than 3 kcal/mole of heat when

it is hydrolyzed

2) The highest-energy phosphate bond in ATP is located between which of the following

groups?

a) Adenosine and phosphate

b) Ribose and phosphate

c) Ribose and adenine

d) Two hydroxyl groups in the ribose ring

e) Two phosphate groups

3) Which of the following is NOT an example of how the energy in ATP is used to do

work?

a) Protein conformational change in skeletal muscle

b) Na+/K+ pump

c) Anabolic protein synthesis

d) Methane synthesis

e) Glycogen synthesis

4) The transfer of phosphate from ATP to glucose occurs in two steps. Given the

following reactions and #G(0)s, what can be said?

Glucose + Pi => glucose-6-P + H20 #G(0) = +3.3 kcal/mole

ATP + H20 => ADP + Pi #G(0) = -7.3 kcal/mole

a) Glucose-6-P will be formed in the initial reaction

b) ATP will transfer to glucose as is due to additive coupling of reactions

c) Heat energy must be added to the initial reaction to make glucose-6-P

d) Only conversation of ATP to ADP will occur

e) The overall #G(0) causes the reaction to work in reverse

5) Reactions in the cell with a positive #G(0) can proceed in the forward direction if the

concentration of ____ is raised to high enough levels, or if the concentration of ____ is

decreased to very low levels.

a) Substrate; Product

b) Product; Substrate

c) Substrate; Enzyme

d) Enzyme; Substrate

e) Product; Enzyme

6) Which of the following best describes thermogenesis?

a) Energy expended for the purpose of conserving heat equal to that expended for

ATP production

b) Energy expended for the purpose of conserving heat in addition to that

expended for ATP production

c) Energy expended for the purpose of generating heat equal to that expended for

ATP production

Page 38: Quiz Biochemistry

Biochemistry 14Mar2009

DO NOT DISTRIBUTE - 38 -

d) Energy expended for the purpose of generating heat in addition to that

expended for ATP production

7.1) Which of the following statements correctly describes reduction of one of the

electron carriers, NAD+ or FAD?

a) NAD+ accepts two electrons as hydrogen atoms to form NADH2

b) NAD+ accepts two electrons that are each donated from a separate atom of the

substrate

c) NAD+ accepts two electrons as a hybrid ion to form NADH

d) FAD releases a proton as it accepts two electrons

e) FAD must accept two electrons at a time

7.2) Our cells spend ATP very rapidly and thus ATP needs to be replenished. Which of

the following (electron-carrying) allows for this replenishing to get more ATP for the

electron transport chain?

a) cAMP

b) FAD

c) ADP

d) Enzymes

e) Coenzymes

20 – Tricarboxylic Acid Cycle

1.1) The starting molecule in the TCA cycles has ____ carbons and the ending molecule

has ____ carbons.

a) 6; 6

b) 5; 5

c) 4; 4

d) 6; 4

e) 4; 6

1.2) What compound follows after isocitrate in the TCA cycle?

a) Citrate

b) "-ketoglutarate

c) Succinyl-CoA

d) Succinate

e) Fumerate

1.3) What compound follows after GDP is converted to GTP in the TCA cycle?

a) Citrate

b) "-ketoglutarate

c) Succinyl-CoA

d) Succinate

e) Oxaloacetate

2) Which of the following describes the products of the TCA cycle?

a) 2CO2 + 4NADH + FADH2 + GTP

b) 3CO2 + 4NADH + FADH2 + 2GTP

c) 2CO2 + 2NADH + FADH2 + GTP

d) CO2 + 3NADH + FADH2 + GTP

e) 2CO2 + 3NADH + FADH2 + GTP

Page 39: Quiz Biochemistry

Biochemistry 14Mar2009

DO NOT DISTRIBUTE - 39 -

3.1) Which of the following coenzymes is unique to "-ketoacid dehydrogenase

complexes?

a) NAD+

b) FAD

c) GDP

d) H2O

e) Lipoic acid

3.2) A patient diagnosed with thiamine deficiency exhibited fatigue and muscle cramps.

The muscle cramps have been related to an accumulation of metabolic acids. Which of

the following metabolic acids is most likely to accumulate in a thiamine deficiency?

a) Isocitric acid

b) Pyruvic acid

c) Succinic acid

d) Malic acid

e) Oxaloacetic acid

3.3) Succinate dehydrogenase differs from all other enzymes in the TCA cycle in that it is

the only enzyme that displays which of the following characteristics?

a) It is embedded in the inner mitochondrial membrane

b) It is inhibited by NADH

c) It contains bound FAD

d) It contains Fe-S centers

e) It is regulated by a kinase

3.4) Coenzyme A is synthesized from which of the following vitamins?

a) Niacin

b) Riboflavin

c) Pantothenate

d) Vitamin A

e) Vitamin C

3.5) Which of the following is NOT a coenzyme required for TCA cycle enzymes?

a) Lipoic acid

b) Riboflavin

c) CoA, FAD, NAD+

d) Niacin

e) Cobalamin

4) How many ATP equivalents (high energy phosphate bonds) are produced in the TCA

cycle per consumption of one acetyl CoA molecule?

a) 3

b) 4

c) 5

d) 8

e) 10

5.1) Regulation of metabolic pathways generally occurs at rate-limiting steps. Both

isocitrate dehydrogenase and "-ketoglutarate dehydrogenase respond directly to changes

in the relative levels of:

a) ATP

b) ADP

Page 40: Quiz Biochemistry

Biochemistry 14Mar2009

DO NOT DISTRIBUTE - 40 -

c) GTP

d) GDP

e) Acetyl CoA

5.2) In contracting heart muscle, what activates isocitrate dehydrogenase and "-

ketoglutarate dehydrogenase?

a) Na+

b) Cl-

c) K+

d) Ca++

e) Mg++

5.3) A child born with pyruvate dehydrogenase complex (PDC) deficiency would likely

present with what type of severe birth complication?

a) Inability to use glucose

b) Ketoacidosis

c) Lactic acidosis

d) Lack of oxaloacetate

e) Inability to use the electron transport chain

5.4) PDC can be rapidly activated though mechanisms involving which of the following?

a) Acetyl CoA

b) NADH

c) ADP

d) Pyruvate

e) Insulin

6) Which of the following is NOT a source of acetyl CoA for the TCA cycle?

a) Decarboxylation of oxaloacetate

b) Degradation of ketone bodies

c) !-oxidation of fatty acids

d) Acetate from the diet

e) Leucine and isoleucine oxidation

21 – Phosphorylation and Mitochondrial Function

1.1) What is the final acceptor in the electron transport chain?

a) NADH

b) FADH+

c) FADH2

d) NAD+

e) O2

1.2) What is the net energy yield of oxidative phosphorylation, in ATP equivalents, per

each mole of FADH2 oxidized?

a) 1.5

b) 2.5

c) 4

d) 5

e) 10

2.1) Which of the following is the only compound of the electron transport chain that is

NOT protein bound?

Page 41: Quiz Biochemistry

Biochemistry 14Mar2009

DO NOT DISTRIBUTE - 41 -

a) NADH dehydrogenase

b) Succinate dehydrogenase

c) Coenzyme Q

d) Cytochrome b-c1 complex

e) Cytochrome c oxidase

2.2) Which of the following is associated with Fe++ reduction (only) as electrons are

accepted during the electron transport chain?

a) NADH dehydrogenase

b) Succinate dehydrogenase

c) Coenzyme Q

d) Cytochrome b-c1 complex

e) Cytochrome c oxidase

2.3) Which of the following involves Cu+ ions facilitating the collection of four electrons

and the reduction of O2 in the electron transport chain?

a) NADH dehydrogenase

b) Succinate dehydrogenase

c) Coenzyme Q

d) Cytochrome b-c1 complex

e) Cytochrome c oxidase

2.4) Which of the following have Fe-S centers to transfer electrons?

a) NADH dehydrogenase and succinate dehydrogenase

b) Succinate dehydrogenase and coenzyme Q

c) Coenzyme Q and cytochrome b-c1 complex

d) Cytochrome b-c1 complex and cytochrome c oxidase

e) Cytochrome c oxidase and NADH dehydrogenase

3) Cyanide poisoning inhibits cytochrome oxidation at which of the following locations

in the electron transport chain?

a) Complex I

b) Complex III

c) Complex IV

d) Coenzyme Q

e) Cytochrome C

4.1) Which of the following would be expected for a patient with an OXPHOS disease?

a) A high ATP:ADP ratio in the mitochondria

b) A high NADH:NAD+ ratio in the mitochondria

c) A deletion of the X chromosome

d) A high activity of complex II of the election transport chain

e) A defect in the integrity of the inner mitochondrial membrane

4.2) Dinitrophenol acts as an uncoupler of oxidative phosphorylation by which of the

following mechanisms?

a) Activating the H+ ATPase

b) Activating coenzyme Q

c) Blocking proton transport across the inner mitochondrial membrane

d) Allowing for proton exchange across the inner mitochondrial membrane

e) The proton motive force will be decreased to a value less than zero

Page 42: Quiz Biochemistry

Biochemistry 14Mar2009

DO NOT DISTRIBUTE - 42 -

4.3) A 25-year-old female presents with chronic fatigue. A series of blood tests are

ordered, and the results suggest that her red blood cell count is low because of iron

deficiency anemia. Such a deficiency would lead to fatigue because of which of the

following?

a) Her decrease in Fe-S centers is impairing the transfer of electrons in the

electron transport chain

b) She is not producing as much H2O in the electron transport chain, leading to

dehydration, which has resulted in fatigue

c) Iron forms a chelate with NADH and FADH2 that is necessary for them to

donate their electrons to the electron transport chain

d) Iron acts as a cofactor for !-ketoglutarate DH in the TCA cycle, a reaction

required for the flow of electrons through the electron transport chain

e) Iron accompanies the protons that are pumped from the mitochondrial matrix to

the cytosolic side of the inner mitochondrial membrane. Without iron, the proton

gradient cannot be maintained to produce adequate ATP.

4.4) What types of DNA or RNA are attacked by OX/PHOS disease?

a) mRNA and tRNA

b) mRNA and rRNA

c) mtDNA and mRNA

d) nDNA and mtDNA

e) nDNA and tRNA

5.1) As ATP is used in the body for energy and ADP levels increase, proton influx

through the ATP synthase increases to create more ATP. This also causes the

electrochemical gradient to ____. But, this happens simultaneously/coupled with the

restoration of the transmembrane electrochemical gradient by increasing proton pumping

and electron flow through the electron transport chain. The result is ____ O2

consumption.

a) Change; No change in

b) Increase; Increased

c) Decrease; Decreased

d) Increase; Decreased

e) Decrease; Increased

5.2) Brown fat thermogenesis works through ____ and is for ____ thermogenesis.

a) Uncoupling; Shivering

b) Coupling; Shivering

c) Uncoupling; Non-shivering

d) Coupling; Non-shivering

6) The inner mitochondrial membrane forms a tight permeability barrier to all polar

molecules, including ATP, ADP, Pi, anions such as pyruvate, and cations such as Ca++,

H+, and K+. ATP-ADP translocase helps transport:

a) ATP and ADP out of the mitochondria

b) ATP and ADP into the mitochondria

c) ATP out of the mitochondria and ADP into the mitochondria

d) ADP out of the mitochondria and ATP into the mitochondria

Page 43: Quiz Biochemistry

Biochemistry 14Mar2009

DO NOT DISTRIBUTE - 43 -

7) If a cell is ischemic, there is no O2 for the electron transport chain. ATP runs

backwards to ADP, making adenine, and opening the mitochondrial permeability

transition pores (MPTPs). What affect would this have on the cell?

a) Increased ATP use

b) Decreased ADP use

c) Decreased oxygen consumption

d) Increased oxygen consumption

e) Inability to create an electron gradient

22 – Generation of ATP from Glucose Glycolysis

1.1) A major role of glycolysis is which of the following?

a) To synthesize glucose

b) To generate energy

c) To produce FADH2

d) To synthesize glycogen

e) To use ATP to generate heat

1.2) Starting with glyceraldehyde 3-phosphate and synthesizing one molecule of

pyruvate, the net yield of ATP and NADH would be which of the following?

a) 1 ATP, 1 NADH

b) 1 ATP, 2 NADH

c) 1 ATP, 4 NADH

d) 2 ATP, 1 NADH

e) 2 ATP, 2 NADH

f) 2 ATP, 4 NADH

g) 3 ATP, 1 NADH

h) 3 ATP, 2 NADH

i) 3 ATP, 4 NADH

1.3) when glycogen is degraded, glucose 1-phosphate is formed. Glucose 1-phosphate

can then be isomerized to glucose 6-phosphate. Starting with glucose 1-phosphate, and

ending with 2 molecules of pyruvate, what is the net yield of glycolysis, in terms of ATP

and NADH formed?

a) 1 ATP, 1 NADH

b) 1 ATP, 2 NADH

c) 1 ATP, 3 NADH

d) 2 ATP, 1 NADH

e) 2 ATP, 2 NADH

f) 2 ATP, 3 NADH

g) 3 ATP, 1 NADH

h) 3 ATP, 2 NADH

i) 3 ATP, 3 NADH

1.4) Which of the following statements correctly describes an aspect of glycolysis?

a) ATP is formed by oxidative phosphorylation

b) 2 ATP are used in the beginning of the pathway

c) Pyruvate kinase is the rate-limiting enzyme

d) One pyruvate and three CO2 are formed from the oxidation of one glucose

molecule

Page 44: Quiz Biochemistry

Biochemistry 14Mar2009

DO NOT DISTRIBUTE - 44 -

e) The reactions take place in the matrix of the mitochondria

1.5) How many moles of ATP are generated by the complete aerobic oxidation of 1 mole

of glucose to 6 moles of CO2?

a) 2-4

b) 10-12

c) 18-22

d) 30-32

e) 60-64

1.6) How many net ATPs are produced in anaerobic glycolysis?

a) 1

b) 2

c) 4

d) 8

e) 12

2) Which of the following enzymes is NOT associated with ATP use or ATP gain?

a) Hexokinase

b) Phosphofructokinase-1

c) Glyceraldehyde 3-phosphate dehydrogenase

d) Phosphoglycerate kinase

e) Pyruvate kinase

3) NADH is produced between which steps of glycolysis?

a) D-Glucose and glucose 6-phosphate

b) Fructose 6-phosphate and fructose 1,6-bisphosphate

c) Glyceraldehyde 3-phosphate and 1,3-bisphosphoglycerate

d) 1,3-bisphosphoglycerate and 3-phosphoglycerate

e) Phosphoenol-pyruvate and pyruvate

4) Which of the following describes anaerobic fates of pyruvate and NADH as they differ

from aerobic glycolysis?

a) Pyruvate is reduced to lactate in the cytosol, thereby using the reducing

equivalents in NADH

b) Pyruvate is reduced to lactate in the cytosol and the reducing equivalents of

NADH enter mitochondria via a shuttle system

c) Pyruvate enters mitochondria and is oxidized to CO2 and H2O, thereby using

the reducing equivalents in NADH

d) Pyruvate enters mitochondria and is oxidized to CO2 and H2O and the

reducing equivalents of NADH enter mitochondria via a shuttle system

5) The eye contains cells that transmit or focus light and these cells cannot, therefore, be

filled with opaque structures. What adaptation is seen to allow for the eye to use ATP and

retain function?

a) Mitochondria are moved to the edges of cells

b) Mitochondria are very thin allowing for increased light transmission

c) Mitochondria are only present at the back of the eye and energy reaches the

front of the eye via simple diffusion

d) Aerobic glycolysis is the main process used

e) Anaerobic glycolysis is the main process used

6) Which of the following best summarizes the Cori Cycle?

Page 45: Quiz Biochemistry

Biochemistry 14Mar2009

DO NOT DISTRIBUTE - 45 -

a) Gluconeogenesis in kidney => Glucose => Muscle use => Lactate production

=> Lactate converted to glucose in kidney

b) Gluconeogenesis in liver => Glucose => Muscle use => Lactate production =>

Lactate converted to glucose in liver

c) Glycolysis in liver => Glucose => Muscle use => Lactate production =>

Lactate converted to glycogen in liver

d) Glycolysis in liver => Glycogen => Muscle use => Lactate production =>

Lactate converted to glucose in liver

e) Gluconeogenesis in liver => Lactate => Muscle use => Glucose production =>

Glucose converted to glycogen in liver

7.1) The major function of phosphofructokinase-1 (PFK-1) and pyruvate dehydrogenase

(PDH) are to respond to feedback indicators at the rate of:

a) ADP utilization

b) ATP utilization

c) NADH utilization

d) Glycogen utilization

e) Lactate utilization

7.2) Which of the following is the rate limiting enzyme of glycolysis?

a) Hexokinase

b) Phosphofructokinase-1

c) Glyceraldehyde 3-phosphate dehydrogenase

d) Phosphoglycerate kinase

e) Pyruvate kinase

7.3) PFK-1 is allosterically activated by 2,6-bisphosphate and which of the following?

a) ATP

b) ADP

c) AMP

d) NAD+

e) FADH2

7.4) Under ischemic conditions, AMP levels within the heart rapidly increase because of

the lack of ATP production via oxidative phosphorylation. The increase in AMP levels

activates an AMP-dependent protein kinase, which phosphorylates the heart isozyme of

____ to activate its kinase activity and start anaerobic glycolysis.

a) Hexokinase

b) PFK-1

c) PFK-2

d) 2,6-bisphosphate

e) Pyruvate kinase

8) If the electron transport chain is interrupted so ATP cannot be produced, glycolysis

operates anaerobically to produce ATP, and lactate is formed eventually leading to lactic

acidosis. Which of the following would NOT lead to lactic acidosis?

a) CO poisoning

b) Cyanide poisoning

c) Ischemia

d) Anoxia

e) A decreased NADH/NAD+ ratio

Page 46: Quiz Biochemistry

Biochemistry 14Mar2009

DO NOT DISTRIBUTE - 46 -

23 – Oxidation of Fatty Acids and Ketone Bodies

1.1) What is the carrier that transports activated long chain fatty acyl groups across the

inner mitochondrial membrane?

a) Acyl CoA

b) Acetyl CoA

c) Carnatine

d) Albumin

e) Fatty acid binding proteins

1.2) What derivative form must fatty acids be activated to before they can participate in

!-oxidation?

a) Acetyl CoA

b) Acyl CoA

c) Carnatine

d) Ketone bodies

e) Polyunsaturated

1.3) An individual with a deficiency of an enzyme in the pathway for carnitine synthesis

is not eating adequate amounts of carnitine in the diet. Which of the following would you

expect during fasting as compared with an individual with an adequate intake and

synthesis of carnitine?

a) Fatty acid oxidation is increased

b) Ketone body synthesis is increased

c) Blood glucose levels are increased

d) The levels of dicarboxylic acids in the blood would be increased

e) The levels of very-long-chain fatty acids in the blood would be increased

2.1) The !-oxidation spiral occurs in the mitochondrial matrix and involves all of the

following steps EXCEPT:

a) Fatty acyl CoA

b) trans #2 Fatty enoyl CoA

c) L-!-Hydroxy acyl CoA

d) !-Keto acyl CoA

e) 1,6-biphosphate CoA

2.2) The oxidation of fatty acids is best described by which of the following sets of

reactions?

a) Oxidation, hydration, oxidation, carbon-carbon bond breaking

b) Oxidation, dehydration, oxidation, carbon-carbon bond breaking

c) Oxidation, hydration, reduction, carbon-carbon bond breaking

d) Oxidation, dehydration, reduction, carbon-carbon bond breaking

e) Reduction, hydration, oxidation, carbon-carbon bond breaking

2.3) At which of the periods listed below will fatty acids be the major source of fuel for

the tissues of the body?

a) Immediately after breakfast

b) Minutes after a snack

c) Immediately after dinner

d) While running the first mile of a marathon

e) While running the last mile of a marathon

Page 47: Quiz Biochemistry

Biochemistry 14Mar2009

DO NOT DISTRIBUTE - 47 -

3.1) What is the result of complete oxidation of palmitate (16 carbons, no double bonds)

to CO2 and water via !-oxidation?

a) 8 FADH2, 8 NADH, 8 acetyl CoA

b) 7 FADH2, 8 NADH, 8 acetyl CoA

c) 8 FADH2, 8 NADH, 7 acetyl CoA

d) 7 FADH2, 7 NADH, 8 acetyl CoA

e) 8 FADH2, 7 NADH, 7 acetyl CoA

3.2) How many ATP molecules are produced from !-oxidation of palmitate?

a) 7 * 2.5 + 7 * 1.5 + 8 * 10 = 108

b) 7 * 2.5 + 7 * 1.5 + 8 * 10 - 2 = 106

c) 8 * 2.5 + 8 * 1.5 + 8 * 10 = 107

d) 8 * 2.5 + 8 * 1.5 + 8 * 10 - 2 = 109

e) 8 * 2.5 + 8 * 1.5 + 8 * 10 + 2 = 111

4) !-oxidation is regulated by the cells’ requirements for energy, or the levels of ____,

because fatty acids cannot be oxidized any faster than ____ are reoxidized in the electron

transport chain.

a) AMP and NAD+; NADH and FADH2

b) ADP and NADH; NADH and FADH2

c) ATP and NADH; NADH and FADH2

d) ATP and NADH; NAD+ and FADH+

e) ADP and NADH; NAD+ and FADH+

5.1) What transformation must occur at double bonds for oxidation of unsaturated fatty

acids to occur?

a) They must be made chiral

b) Hydrolysis must occur first

c) Trans bonds are turned into cis bonds

d) Cis bonds are turned into trans bonds

e) Reduction takes place initially before hydrolysis

5.2) Fatty acids containing an odd number of carbons undergo !-oxidation until the last

spiral. How many carbons are left in the fatty acyl CoA in the last spiral of !-oxidation?

a) 1

b) 2

c) 3

d) 4

e) 5

5.3) Where are very-long-chain fatty acids (24 to 26 carbons) oxidized in chain-

shortening pathways that use FAD-containing oxidase to allow for !-oxidation?

a) Mitochondria

b) Lysosomes

c) Peroxisomes

d) Cytoplasm

e) Endoplasmic reticulum

6) %-oxidation of fatty acids uses enzymes from the endoplasmic reticulum. What

product is created by this process, allowing for the continuation of !-oxidation?

a) Carboxylic acids

b) Dicarboxylic acids

Page 48: Quiz Biochemistry

Biochemistry 14Mar2009

DO NOT DISTRIBUTE - 48 -

c) Tricarboxilic acids

d) Oligocarboxylic acids

e) Polycarboxylic acids

7.1) Which of the following !-oxidation enzyme reactions is reversible, allowing for the

production of ketone bodies in the liver?

a) Acyl CoA dehydrogenase

b) Enoyl CoA hydratase

c) !-hydroxyl acyl CoA dehydrogenase

d) !-keto thiolase

7.2) Which of the following is NOT a ketone body?

a) Acetoacetate

b) Acetyl CoA

c) D-!-Hydroxybutyrate

d) Acetone

7.3) !-Hydroxybutyrate is oxidized to acetoacetate, which is activated by accepting a

CoA group from what part of the TCA cycle?

a) Acetoacetyl CoA

b) Acetyl CoA

c) Succinyl CoA

d) Succinate

e) Oxaloacetate

7.4) Which of the following ketogenic amino acids can form acetyl CoA and acetoacetyl

CoA in the liver as well as other extra-hepatic tissues?

a) Leucine

b) Lysine

c) Tryptophan

d) Tyrosine

e) Phenylalanine

8) Patients with type I diabetes mellitus have a deficiency in insulin and an excess of

glucagon. This leads to excess fat breakdown and ketone body production in the liver,

leading to ketoacidosis. Insulin normally ____ acetyl CoA carboxylase and ____ !-

oxidation.

a) Stimulates; Stimulates

b) Inhibits; Inhibits

c) Stimulates; Inhibits

d) Inhibits; Stimulates

9) Medium-chain acyl-CoA dehydrogenase (MCAD) deficiency is a disorder involving

the substitution of a T for an A in the MCAD gene. This results in fatty acids only being

oxidize down to the medium-chain length. What type of diet would these patients need to

maintain a healthy state?

a) High-protein and avoidance of prolonged fasting

b) High-protein and avoidance of over-eating

c) High-carbohydrate and avoidance of prolonged fasting

d) High-carbohydrate and avoidance of over-eating

e) High-fat and avoidance of prolonged fasting

Page 49: Quiz Biochemistry

Biochemistry 14Mar2009

DO NOT DISTRIBUTE - 49 -

24 – Oxygen Toxicity and Free Radical Injury

1) Free radicals are capable of independent existence and have a single unpaired electron.

Which of the following best describes a reactive oxygen species (ROS)?

a) Free radicals capable of interacting with oxygen

b) Free radicals that cannot interact with oxygen

c) Oxygen compounds that are reactive free radicals

d) Oxygen compounds that can only react with free radicals

e) Oxygen compounds that cannot react with free radicals

2) Which of the following is the most reactive free radical in the body?

a) OH•

b) O2-

c) H2O2

d) RCOO•

e) ONOO-

3.1) Which of the following is NOT a disease state associated with free radical injury?

a) Aging

b) Parkinson disease

c) Alzheimer disease

d) Myocardial infarction

e) Diabetes

3.2) An accumulation of hydrogen peroxide in a cellular compartment can be converted

to dangerous radical forms in the presence of which metal?

a) Se

b) Fe

c) Mn

d) Mg

e) Mb

4.1) ROS are constantly being formed in the cell. Which of the following is a major site

of superoxide generation?

a) Citrate

b) Oxaloacetate

c) Carnitine

d) Acetyl CoA

e) Coenzyme Q

4.2) Superoxide dismutase catalyzes which of the following reactions?

a) O2- + e- + 2H+ yields H2O2

b) 2O2- + 2H+ yields H2O2 + O2

c) O2- + HO• + H+ yields CO2 + H2O

d) H2O2 + O2 yields 4H2O

e) O2- + H2O2 + H+ yields 2H2O + O2

5.1) All of the following cellular components may be damaged by free radicals EXCEPT:

a) Lipids

b) Proteins

c) Carbohydrates

d) DNA

e) Alcohol

Page 50: Quiz Biochemistry

Biochemistry 14Mar2009

DO NOT DISTRIBUTE - 50 -

5.2) The level of oxidative damage to mitochondrial DNA is 10 times greater than that of

nuclear DNA. This could be due, in part, to which of the following?

a) Superoxide dismutase is present in the mitochondria

b) The nucleus lacks glutathione

c) The nuclear membrane presents a barrier to reactive oxygen species

d) The mitochondrial membrane is permeable to reactive oxygen species

e) Mitochondrial DNA lacks histones

6.1) During an immune response (neutrophils, eosinophils, monocytes), what accounts

for the formation of ROS?

a) Glucose use increased

b) Oxygen use increased

c) ATP use increased

d) ADP use increased

e) Nitrogen use increased

6.2) During an inflammatory response, what reacts with NO to create RNOS when

neutrophils produce excess NO?

a) Superoxide

b) Hydrogen peroxide

c) Hydroxyl

d) Peroxyl

e) Peroxynitrite

7.1) Premature infants requiring oxygen therapy are usually given what vitamin, which

along with SOD can help prevent retinopathy and subsequent blindness?

a) A

b) C

c) D

d) E

e) K

7.2) Which of the following has been shown by current (2007) epidemiologic evidence

that it is effective as a supplement on top of normal dietary intake?

a) !-carotene

b) Vitamin C

c) Vitamin E

d) Vitamin K

e) Vitamin B6

7.3) Which of the following vitamins or enzymes is unable to protect against free radical

damage?

a) !-carotene

b) Glutathione peroxidase

c) Superoxide dismutase

d) Vitamin B6

e) Vitamin C

f) Vitamin E

7.4) The mechanism of vitamin E as an antioxidant is best described by which of the

following?

Page 51: Quiz Biochemistry

Biochemistry 14Mar2009

DO NOT DISTRIBUTE - 51 -

a) Vitamin E binds to free radicals and sequesters them from the contents of the

cell

b) Vitamin E participates in the oxidation of the radicals

c) Vitamin E participates in the reduction of the radicals

d) Vitamin E forms a covalent bond with the radicals, thereby stabilizing the

radical state

e) Vitamin E inhibits the enzymes that produce free radicals

25 – Metabolism of Ethanol

1.1) Alcohol (ethanol) is metabolized in the liver by alcohol dehydrogenase (ADH) with

reduction of NAD+ to NADH. What is the end product of the metabolic pathway?

a) Acetaldehyde

b) Acetate

c) Acetone

d) Methanol

e) Ether

1.2) The fate of the product of ethanol metabolism is which of the following?

a) It is taken up by other tissues and activated to acetyl CoA

b) It is toxic to the tissues of the body and can lead to hepatic necrosis

c) It is excreted in bile

d) It enters the TCA cycle directly to be oxidized

e) It is converted into NADH by alcohol dehydrogenase

2.1) The microsomal ethanol oxidizing system (MEOS) accounts for a small percentage

of ethanol oxidation. What is the first product created by MEOS with the oxidation of

NADPH to NADP+?

a) Acetaldehyde

b) Acetate

c) Acetone

d) Methanol

e) Ether

2.2) Induction of CYP2E1 would result in which of the following?

a) A decreased clearance of ethanol from the blood

b) A decrease in the rate of acetaldehyde production

c) A low possibility of the generation of free radicals

d) Protection from hepatic damage

e) An increase of one’s alcohol tolerance level

3) Phenobarbital is converted to an inactive metabolite by cytochrome P450

monooxygenases CYP2B1 and CYP2B2. What would likely happen if a high dose of

phenobarbital was given and then excess ethanol was consumed?

a) A higher alcohol tolerance would be seen

b) A lower alcohol tolerance would be seen

c) Phenobarbital would have less of an effect

d) Phenobarbital would accumulate in the blood

e) Phenobarbital would be immediately inactivated

4) What is the maximum total energy yield in moles of ATP per mole of ethanol?

a) 5

Page 52: Quiz Biochemistry

Biochemistry 14Mar2009

DO NOT DISTRIBUTE - 52 -

b) 8

c) 10

d) 11

e) 13

5.1) Many of the acute affects of ethanol ingestion arise from the increase in what ratio in

the liver?

a) ATP/ADP

b) AMP/ATP

c) NAD+/NADH

d) NADH/NAD+

e) FADH2/FADH

5.2) Which of the following would be expected to occur after acute alcohol ingestion?

a) The activation of fatty acid oxidation

b) Lactic acidosis

c) The inhibition of ketogenesis

d) An increase in the NAD+/NADH ratio

e) An increase in gluconeogenesis

5.3) A chronic alcoholic is in treatment for alcohol abuse. The drug disulfiram is

prescribed for the patient. This drug deters the consumption of alcohol by which of the

following mechanisms?

a) Inhibiting the absorption of ethanol so that an individual cannot become

intoxicated, regardless of how much they drink

b) Inhibiting the conversion of ethanol to acetaldehyde, which would cause the

excretion of unmetabolized ethanol

c) Blocking the conversion of acetaldehyde to acetate, which would cause the

accumulation of acetaldehyde

d) Activating the excessive metabolism of ethanol to acetate, which causes

inebriation with consumption of a small amount of alcohol

e) Preventing the excretion of acetate, which causes nausea and vomiting

6) Which one of the following consequences of chronic alcohol consumption is

irreversible?

a) Inhibition of fatty acid oxidation

b) Activation of triacylglycerols synthesis

c) Ketoacidosis and lactic acidosis

d) Hyperuricemia and hypoglycemia

e) Liver cirrhosis

7) Acetaldehyde toxicity causes a build-up of proteins in the liver. Which of the

following is a clinical result of this toxicity?

a) Hepatitis

b) Hypoglycemia

c) Liver cirrhosis

d) Ketoacidosis

e) Lactic acidosis

26 – Basic Concepts in the Regulation of Fuel Metabolism by Insulin, Glucagon

1.1) Which of the following is NOT true when insulin is released?

Page 53: Quiz Biochemistry

Biochemistry 14Mar2009

DO NOT DISTRIBUTE - 53 -

a) Blood glucose is down

b) Glycogen synthesis increases

c) Fatty acid synthesis increases

d) Triglyceride synthesis increases

e) Liver glycolysis increases

1.2) A patient with type I diabetes mellitus takes an insulin injection before eating dinner

then gets distracted and does not eat. Approximately 3 hours later, the patient becomes

shaky, sweaty, and confused. These symptoms have occurred because of which of the

following?

a) Increased glucagon release from the pancreas

b) Decreased glucagon release from the pancreas

c) High blood glucose levels

d) Low blood glucose levels

e) Elevated ketone body levels

2.1) Which of the following has the highest demand for glucose as a fuel?

a) Brain

b) Muscle

c) Heart

d) Liver

e) Pancreas

2.2) Insulin is created as a prehormone that is converted to proinsulin in the RER.

Glucose enters the ____ cells (islets of Langerhans) and causes a(n) ____ in ATP levels.

K+ channels are inhibited and Ca++ channels open, stimulating insulin secretion.

a) "; Decrease

b) "; Increase

c) !; Decrease

d) !; Increase

2.3) Which of the following is a negative regulator of glucagon release?

a) Gut hormones

b) Cortisol

c) Glucose

d) Amino acids

e) Epinephrine

3) Caffeine is a potent inhibitor of the enzyme cAMP phosphodiesterase. Which of the

following consequences would you expect to occur in the liver after drinking two cups of

strong espresso coffee?

a) A prolonged response to insulin

b) A prolonged response to glucagon

c) An inhibition of protein kinase A

d) An enhancement of glycolytic activity

e) A reduced rate of glucose export to the circulation

4) Which of the following is a negative regulator of insulin release?

a) Glucose

b) Amino acids

c) Neural input

d) Gut hormones

Page 54: Quiz Biochemistry

Biochemistry 14Mar2009

DO NOT DISTRIBUTE - 54 -

e) Epinephrine

5.1) Glucagon release does not alter muscle metabolism because of which of the

following?

a) Muscle cells lack adenylate cyclase

b) Muscle cells lack protein kinase A

c) Muscle cells lack G proteins

d) Muscle cells lack GTP

e) Muscle cells lack the glucagon receptor

5.2) Insulin binds to the ____ subunit of the insulin receptor and has ____ activity.

a) "; Serine/Threonine

b) "; Tyrosine kinase

c) !; Serine/Threonine

d) !; Tyrosine kinase

5.3) The glucagon receptor is coupled to production of which of the following?

a) cAMP

b) cGMP

c) Ca++

d) Calmodulin

e) PKC

5.4) Which of the following is NOT a signal transduction principle by which glucagon

works?

a) Specificity of action in tissues is conferred by the receptor on a target cell

b) Signal transduction involves amplification of the first message

c) Signals terminate slowly

d) Metabolic responses are integrated

e) Signals can be augmented or antagonized

27 – Digestion, Absorption and Transport of Carbohydrates

1.1) Lactose is a disaccharide composed of ____ and sucrose is a disaccharide composed

of ____.

a) Glucose and galactose; Glucose and fructose

b) Glucose and fructose; Glucose and galactose

c) Glucose and maltose; Glucose and fructose

d) Glucose and fructose; Glucose and maltose

e) Glucose and maltose; Glucose and dextrose

1.2) What causes the initial conversion of large polysaccharides to "-dextrins?

a) Salivary "-amylase

b) Pancreatic "-amylase

c) Trehalase

d) !-glycosidase

e) !-glucoamylase

1.3) After digestion of a piece of cake that contains flour, milk, and sucrose as its primary

ingredients, the major carbohydrate products entering the blood are which of the

following?

a) Glucose

b) Fructose and galactose

Page 55: Quiz Biochemistry

Biochemistry 14Mar2009

DO NOT DISTRIBUTE - 55 -

c) Galactose and glucose

d) Fructose and glucose

e) Glucose, galactose, and fructose

2.1) An alcoholic patient develops a pancreatitis that affected his exocrine pancreatic

function. He exhibited discomfort after eating a high-carbohydrate meal. The patient

most likely had a reduced ability to digest which of the following?

a) Starch

b) Lactose

c) Fiber

d) Sucrose

e) Maltose

2.2) What enzymatic defect is observed with lactose intolerance?

a) Maltase

b) !-glycosidase

c) !-glucoamylase

d) Lactase

e) Trehalase

3) Although beneficial effects from fiber are related to the specific type of fiber, which of

the following is true regarding beneficial effects of insoluble (non soluble) fiber? (current

research in 2007)

a) Lower blood cholesterol for general public

b) Slow the rate of glucose absorption for diabetes patients

c) Softens stool for colonic disease patients

d) Reducing bile acid resorption in the small intestine

4.1) A patient has a genetic defect that causes intestinal epithelial cells to produce

disaccharidases of much lower activity than normal. Compared with a normal person,

after eating a bowl of milk and oatmeal sweetened with table sugar, this patient will

exhibit higher levels of which of the following?

a) Maltose, sucrose, and lactose in the stool

b) Starch in the stool

c) Galactose and fructose in the blood

d) Glycogen in the muscles

e) Insulin in the blood

4.2) Which of the following ion-glucose cotransporters is necessary to move galactose

(and glucose partially) from the intestinal lumen to cells along with fructose?

a) K+

b) Na+

c) Ca++

d) Mg++

e) Cl-

5.1) A type I diabetic neglects to take his insulin injections while on a weekend vacation.

Cells of which tissue would be most greatly affected by this mistake?

a) Brain

b) Liver

c) Muscle

d) Red blood cells

Page 56: Quiz Biochemistry

Biochemistry 14Mar2009

DO NOT DISTRIBUTE - 56 -

e) Pancreas

5.2) Insulin-dependent glucose uptake occurring in skeletal muscle and adipose tissue

uses what transporter?

a) GLUT 1

b) GLUT 2

c) GLUT 3

d) GLUT 4

e) GLUT 5

6.1) The facilitative transport most responsible for transporting fructose from the blood

into cells is which of the following?

a) GLUT 1

b) GLUT 2

c) GLUT 3

d) GLUT 4

e) GLUT 5

6.2) What non-insulin-dependent glucose transporter is associated with human

erythrocytes and the blood-brain barrier?

a) GLUT 1

b) GLUT 2

c) GLUT 3

d) GLUT 4

e) GLUT 5

6.3) What non-insulin-dependent glucose transporter is associated with neurons?

a) GLUT 1

b) GLUT 2

c) GLUT 3

d) GLUT 4

e) GLUT 5

7) Which of the following is NOT different between neural and non-neural transport of

glucose with regard to cellular structure?

a) Neural endothelial cells have tight junctions

b) Neural intercellular space is narrow

c) Neural cells do not do pinocytosis

d) Neural cells have a discontinuous basement membrane

e) Neural cells have glucose transporters in both membranes

28 – Formation & Degradation of Glycogen

1) What is glycogen initially broken down to in the liver and muscle?

a) Glucose-1-P

b) Glucose-2-P

c) Glucose-4-P

d) Glucose-6-P

e) Glucose

2) In which of the following tissues does glycogen (more precisely glucose-6-P) serve as

a source of blood glucose instead of a source of ATP generation?

a) Muscle

Page 57: Quiz Biochemistry

Biochemistry 14Mar2009

DO NOT DISTRIBUTE - 57 -

b) Kidney

c) Liver

d) Brain

e) Heart

3.1) Which of the following is NOT a benefit of branching in glycogen?

a) Increased sites for synthesis

b) Increased sites for degradation

c) Increased solubility

d) Increased resistance to 4:6 transferase

3.2) In a glucose tolerance test, an individual in the basal metabolic state ingests a large

amount of glucose. If the individual is normal, this ingestion should result in which of the

following?

a) An enhanced glycogen synthase activity in the liver

b) An increased ratio of glycogen phosphorylase A to glycogen phosphorylase B

in the liver

c) An increased rate of lactate formation by red blood cells

d) An inhibition of protein phosphatase I activity in the liver

e) An increase of cAMP levels in the liver

3.3) Consider a type I diabetic who has neglected to take insulin for the past 72 hours and

has not eaten much as well. Which of the following best describes the activity level of

hepatic enzymes involved in glycogen metabolism under these conditions?

Glycogen Phosphorylase Glycogen

Synthase Kinase Phosphorylase

a) Active Active Active

b) Active Active Inactive

c) Active Inactive Inactive

d) Inactive Inactive Inactive

e) Inactive Active Inactive

f) Inactive Active Active

3.4) Glycogen synthase is the enzyme that attaches the glucosyl residues in what type of

bonds (regulated step in the pathway) during glycogen synthesis?

a) "-1,6-glycosidic

b) "-1,4-glycosidic

c) !-1,6-glycosidic

d) !-1,4-glycosidic

4.1) The enzyme glycogen phosphorylase starts at the end of a chain and successively

cleaves glucosyl residues by adding phosphate to the terminal glycosidic bond during

glycogen degradation. Why must a debrancher enzyme be used?

a) Because glycogen phosphorylase cannot cleave hydrogen bonds

b) Because glycogen phosphorylase cannot cleave covalent bonds

c) Because glycogen phosphorylase cannot cleave use UDP-glucose

d) Because glycogen phosphorylase does not have enough energy

e) Because glycogen phosphorylase is sterically hindered

4.2) The degradation of glycogen normally produces which of the following?

a) More glucose than glucose 1-phosphate

b) More glucose 1-phosphate than glucose

Page 58: Quiz Biochemistry

Biochemistry 14Mar2009

DO NOT DISTRIBUTE - 58 -

c) Equal amounts of glucose and glucose 1-phosphate

d) Neither glucose or glucose 1-phosphate

e) Only glucose 1-phosphate

5.1) A patient has large deposits of liver glycogen, which, after an overnight fast, had

shorter than normal branches. This abnormality could be caused by a defective form of

which of the following proteins or activities?

a) Glycogen phosphorylase

b) Glucagon receptor

c) Glycogenin

d) Amylo 1,6 glucosidase

e) Amylo 4,6 transferase

5.2) An adolescent patient with a deficiency of muscle phosphorylase was examined

while exercising her forearm by squeezing a rubber ball. Compared with a normal person

performing the same exercise, this patient would exhibit which of the following?

a) Exercise for a longer time without fatigue

b) Have increased glucose levels in blood drawn from her forearm

c) Have decreased lactate levels in blood drawn form her forearm

d) Have lower levels of glycogen in biopsy specimens from her forearm muscle

e) Hyperglycemia

5.3) An adolescent patient presents with a lysosomal "-glucosidase disorder (glycogen

storage disorder). Which of the following would most likely be seen?

a) Increased glucose deposits in lysosomes

b) Decreased glucose deposits in lysosomes

c) Increased glycogen deposits in lysosomes

d) Decreased glycogen deposits in lysosomes

e) Increased fat deposits in lysosomes

6.1) During regulation of glycogen synthesis/degradation in the liver, phosphorylase

kinase adds a phosphate to specific ____ residues on glycogen phosphorylase ____,

thereby converting it to the active glycogen phosphorylase ____.

a) Serine; A; B

b) Threonine; A; B

c) Serine; B; A

d) Threonine; B; A

6.2) During regulation of glycogen synthesis/degradation in the liver, glycogen is

degraded to glucose 1-phosphate as a result of ____ of glycogen synthase and the ____ of

glycogen phosphorylase.

a) Inhibition; Activation

b) Activation; Inhibition

c) Inhibition; Inhibition

d) Activation; Activation

7.1) The formation of cAMP and the resultant activation of protein kinase A are initiated

by the binding of what to plasma membrane receptors (glycogenolysis)?

a) Atropine

b) Epinephrine

c) Adenosine

d) Glucagon

Page 59: Quiz Biochemistry

Biochemistry 14Mar2009

DO NOT DISTRIBUTE - 59 -

e) Insulin

7.2) After muscle contraction, what is created that allosterically activates glycogen

phosphorylase b?

a) ATP

b) ADP

c) AMP

d) UDP-glucose

e) Glycogen phosphorylase a

29 – Pathways of Sugar Metabolism

1.1) Fructose enters epithelial cells and other types of cells by facilitated diffusion on

which transporter?

a) GLUT 1

b) GLUT 2

c) GLUT 3

d) GLUT 4

e) GLUT 5

1.2) What are the two glycolysis intermediates that fructose is metabolized to?

a) Fructose-1-P and Glyceraldehyde

b) Glyceraldehyde and Glyceraldehyde-3-P

c) Glyceraldehyde-3-P and Dihydroxyacetone-P

d) Dihydroxyacetone-P and Glucose-6-P

e) Glucose-6-P and Glucose-1-P

2.1) An alcoholic is brought to the Emergency Room in a hypoglycemic coma. Since

alcoholics are frequently malnourished, which of the following enzymes can be used to

test for a thiamine deficiency?

a) Aldolase

b) Transaldolase

c) Transketolase

d) Glucose 6-phosphate dehydrogenase

e) UDP-galactose epimerase

2.2) Deficiency in fructokinase is a benign problem. However, genetic aldolase B

deficiency is life threatening because of the toxic accumulation of which of the following,

leading to hypoglycemia?

a) Fructose

b) Sorbitol (polyol)

c) Fructose 6-phosphate

d) Fructose 1-phosphate

e) Glyceraldehyde 3-phosphate

3.1) Intravenous fructose feeding can lead to lactic acidosis caused by which of the

following?

a) Bypassing the regulated pyruvate kinase step

b) Bypassing the regulated PFK-1 step

c) Allosterically activating aldolase B

d) Allosterically activating lactate dehydrogenase

e) Increasing the [ATP]/[ADP] ratio in the liver

Page 60: Quiz Biochemistry

Biochemistry 14Mar2009

DO NOT DISTRIBUTE - 60 -

3.2) The polyol pathway is the conversion of?

a) Sorbitol to glucose

b) Fructose to glucose

c) Fructose to galactose

d) Glucose to galactose

e) Glucose to fructose

4) A three-week-old child is being seen for early cataracts forming in her eyes. The

physician believes the problem is hypergalactosemia and orders a test for the deficient

enzyme, which is:

a) Galactose-1-P uridylytransferase

b) Galactokinase

c) UDP-glucose epimerase

d) Sorbitol dehydrogenase

e) Aldose reductase

5) The pentose phosphate pathway yields what products, the first being used in all cells

for reductive detoxification and the second being used in most cells for nucleotide

synthesis?

a) Glucose-6-P and glyceraldehyde-3-P

b) Glyceraldehyde-3-P and glucose-6-P

c) NADPH and ribose-5-P

d) Ribose-5-P and NADPH

e) ATP and ADP

6) What other products are created in the pentose phosphate pathway, the first being part

of the oxidative pathway and the second part of the non-oxidative pathway?

a) Glucose-6-P and glyceraldehyde-3-P

b) Glyceraldehyde-3-P and Glucose-6-P

c) NADPH and ribose-5-P

d) Ribose-5-P and NADPH

e) ATP and ADP

7) A glucose 6-phosphate dehydrogenase deficiency leads to compromise of the

glutathione defense system. How does this affect red blood cells?

a) Hemoglobin will only hold one oxygen molecule

b) Hemoglobin cannot hold oxygen

c) The cells will shrink

d) The cells will lyse due to influx of glucose

e) The cells will lyse to due mechanical stress via Heinz bodies

30 – Synthesis of Glycosides, Lactose, Glycoproteins, Glycolipids

1.1) What is a common starting point for many derived sugars and oligosaccharides?

a) UDP-glucuronate

b) Glucoronide

c) Glycogen

d) Lactose

e) UDP-glucose

1.2) The immediate carbohydrate precursor for glycolipid and glycoprotein synthesis are

which of the following?

Page 61: Quiz Biochemistry

Biochemistry 14Mar2009

DO NOT DISTRIBUTE - 61 -

a) Sugar phosphates

b) Sugar acids

c) Sugar alcohols

d) Nucleotide sugars

e) Acyl-sugars

1.3) A newborn is diagnosed with neonatal jaundice. In this patient, the bilirubin

produced lacks which of the following carbohydrates?

a) Glucose

b) Gluconate

c) Glucuronate

d) Galactose

e) Galactitol

1.4) The nitrogen donor for the formation of amino sugars in which of the following?

a) Ammonia

b) Asparagine

c) Glutamine

d) Adenine

e) Dolichol

2.1) Which of the following best describes a mother with galactosemia caused by a

deficiency of galactose 1-phosphate uridylyl transferase?

a) She can convert galactose to UDP-galactose for lactose synthesis during

lactation

b) She can form galactose 1-phosphate from galactose

c) She can use galactose as a precursor to glucose production

d) She can use galactose to produce glycogen

e) She will have lower than normal levels of serum galactose after drinking milk

2.2) NAD+ dependent dehydrogenase oxidizes the alcohol on what carbon of glucose to

form glucuronate?

a) C1

b) C2

c) C4

d) C5

e) C6

2.3) How do glucuronides increase the solubility of and cause the excretion of bilirubin,

drugs, xenobiotics, and other compounds?

a) Add a positive charge to compounds with a hydrogen group

b) Add a positive charge to compounds with a hydroxyl group

c) Add a negative charge to compounds with a hydrogen group

d) Add a negative charge to compounds with a hydroxyl group

3) Which of the following is NOT a metabolic route of UDP-glucuronate?

a) UDP-glucose

b) Iduronate

c) Proteoglycans and glycoproteins

d) UDP-xylose

e) Glucuronides

Page 62: Quiz Biochemistry

Biochemistry 14Mar2009

DO NOT DISTRIBUTE - 62 -

4) The indirect form of bilirubin is bound to albumin in the blood. The direct form, also

known as the ____ form, is ____ soluble.

a) Conjugated; Water

b) Conjugated; Lipid

c) Non-conjugated; Water

d) Non-conjugated; Lipid

5.1) How does UDP-galactose form from UDP-glucose?

a) Hydrolysis

b) Peptide bonding

c) Epimerization

d) Covalent bonding

e) Reduction

5.2) What modifier protein is synthesized after parturition in response to the hormone

prolactin?

a) "-lactalbumin

b) !-lactalbumin

c) Galactosyltransferase

d) UDP-galactose

e) UDP-glucose

5.3) Which of the following correctly lists the steps in lactose synthesis?

a) Glucose-1-P => UDP-galactose => UDP-glucose => Lactose

b) Glucose-1-P => UDP-glucose => UDP-galactose => Lactose

c) Fructose-1-P => UDP-galactose => UDP-glucose => Lactose

d) Fructose-1-P => UDP-glucose => UDP-galactose => Lactose

e) Fructose-1-P => UDP-fructose => UDP-galactose => Lactose

6.1) What amino acids are used for “O-linkages” for glycoproteins?

a) Serine/Threonine

b) Cysteine/Threonine

c) Alanine/Arginine

d) Arginine/Tryptophan

e) Asparagine/Serine

6.2) Which of the following is the most common precursor for the addition of sugars that

are usually found in glycoproteins?

a) UDP-sugars

b) GTP-sugars

c) CMP-NANA

d) L-fructose

e) Mannose

6.3) What amino acid is used for “N-linkages” for glycoproteins?

a) Lysine

b) Cysteine

c) Alanine

d) Arginine

e) Asparagine

Page 63: Quiz Biochemistry

Biochemistry 14Mar2009

DO NOT DISTRIBUTE - 63 -

7.1) Some glycoproteins are produced like secretory proteins, but hydrophobic regions of

the protein remain attached to the cell membrane, and the carbohydrate portion extends

into the extracellular space. These serve as receptors for all of the following EXCEPT:

a) Hormones

b) Transport proteins

c) Cell attachment sites

d) Lysosomal enzymes

e) Bacterial/viral binding sites

7.2) Which of the following helps with cell trafficking when attached to proteins?

a) Glycoproteins

b) Glycolipids

c) Amino acids

d) Cholesterol

e) Second messengers

8) I-cell (inclusion cell) disease is caused by a deficiency in phosphotransferase in the

Golgi apparatus. Normally, this tags cells with Mannose-6-P to be sent to the:

a) RER

b) SER

c) Peroxisomes

d) Lysosomes

e) Nucleus

9.1) Which of the following glycolipids would accumulate in a patient with Sandhoff

disease?

a) GM1

b) Lactosyl-ceramide

c) Globoside

d) Glucocerebroside

e) CM3

9.2) Glycolipids are a derivative of what lipid?

a) Cholesterol

b) Phosphosine

c) Sphingosine

d) Wax (fatty acid and alcohol)

e) Triacyglycerol

10.1) Type A blood contains which of the following at the molecular nonreducing end?

a) Gal

b) GalNAc

c) Gal and GalNAc

d) Neither Gal or GalNAc

10.2) A patient with red blood type B could receive blood from:

a) Type O

b) Type A

c) Type B

d) Types O and A

e) Types O and B

11) What causes lysosomal storage disorder?

Page 64: Quiz Biochemistry

Biochemistry 14Mar2009

DO NOT DISTRIBUTE - 64 -

a) Inadequate degradation of sphingolipids

b) Inadequate degradation of sphingoproteins

c) Complete degradation of sphingolipids

d) Complete degradation of sphingoproteins

12.1) Which of the following is NOT characteristic of infant onset Tay-Sach disease?

a) Diabetes

b) Paralysis

c) Dementia

d) Blindness

e) Developmental retardation

12.2) In Tay-Sach disease (and other gangliosidoses like Fabry and Gaucher disease),

what accumulates in excess due to poorly active lysosomal enzyme function?

a) Sugars

b) Lipids

c) Amino acids

d) Proteins

e) Alcohols

31 – Gluconeogenesis and Maintenance of Blood Glucose Levels

1.1) A common intermediate in the conversion of glycerol and lactate to glucose is which

of the following?

a) Pyruvate

b) Oxaloacetate

c) Malate

d) Glucose 6-phosphate

e) Phosphoenolpyruvate

1.2) A patient presented with a bacterial infection that produced an endotoxin that inhibits

phosphoenolpyruvate carboxykinase. In this patient, then, under these conditions, glucose

production from which of the following precursors would be inhibited?

a) Alanine

b) Glycerol

c) Even-chain-number fatty acids

d) Phosphoenolpyruvate

e) Galactose

1.3) Which of the following statements best describes glucagon?

a) It acts as an anabolic hormone

b) It acts on skeletal muscle, liver, and adipose tissue

c) It acts primarily on the liver and adipose tissue

d) Its concentration in the blood increases after a high-carbohydrate meal

e) Its concentration increases in the blood when insulin levels increase

1.4) Which of the following is most likely to occur in a normal individual after ingesting

a high-carbohydrate meal?

a) Only insulin levels decrease

b) Only insulin levels increase

c) Only glucagon levels increase

d) Both insulin and glucagon levels decrease

Page 65: Quiz Biochemistry

Biochemistry 14Mar2009

DO NOT DISTRIBUTE - 65 -

e) Both insulin and glucagon levels increase

1.5) A patient arrives at the hospital in an ambulance. She is currently in a coma. Before

lapsing into the coma, her symptoms included vomiting, dehydration, low blood pressure,

and a rapid heartbeat. She also had relatively rapid respirations, resulting in more carbon

dioxide exhaled. These symptoms are consistent with which of the following conditions?

a) The patient lacks a pancreas

b) Ketoalkolosis

c) Hypoglycemic coma

d) Diabetic ketoacidosis

e) Insulin shock in a diabetic patient

1.6) How many reaction steps differ between glycolysis and gluconeogenesis?

a) 1

b) 2

c) 3

d) 4

e) 5

2) Which of the following enzymes needed for gluconeogenesis is NOT part of the steps

that are irreversible in glycolysis?

a) Pyruvate carboxylase

b) PEP carboxykinase

c) Fructose bisphosphatase

d) Glucose 6-phosphatase

3) Which of the following tissues requires gluconeogenesis during fasting?

a) Red blood cells

b) Liver

c) Skeletal muscle

d) Smooth muscle

e) Cardiac muscle

4) In the fasting state (in the liver) glycerol, amino acids, and lactate combine with ____

to form glucose and in the starved state they combine with ____ to form glucose.

a) Glucose; Glucose

b) Glycogen; Glycogen

c) Glucose; Glycogen

d) Glycogen; Glucose

e) Glucose; ATP

5) Glucose from gluconeogenesis in the kidney is mostly used by:

a) Skeletal muscle

b) Red blood cells

c) The liver

d) The heart

e) The kidney

6.1) Which of the following gluconeogenic conversions requires biotin and refills

(anaplerotic) the oxaloacetate that is used for the synthesis of glucose?

a) Pyruvate to phosphoenolpyruvate

b) Phosphoenolpyruvate to fructose 1,6-bisphosphate

c) Fructose 1,6-bisphosphate to fructose 6-phosphate

Page 66: Quiz Biochemistry

Biochemistry 14Mar2009

DO NOT DISTRIBUTE - 66 -

d) Glucose 6-phosphate to glucose

6.2) Which of the following gluconeogenic conversions does NOT occur in the cytosol?

a) Pyruvate to phosphoenolpyruvate

b) Phosphoenolpyruvate to fructose 1,6-bisphosphate

c) Fructose 1,6,-bisphosphate to fructose 6-phosphate

d) Glucose 6-phosphate to glucose

7) Which of the following can NOT be converted to glucose?

a) Lactate

b) Tryptophan

c) Glycerol

d) Alanine

8) What enzyme is involved in the Cori Cycle (involving red blood cells)?

a) Glycerol kinase

b) Glycerol 3-phosphate dehydrogenase

c) Lactate dehydrogenase

d) Alanine aminotransferase

e) Phosphoenolpyruvate carboxykinase

9) Which of the following is NOT needed for the gluconeogenesis conversion of pyruvate

to oxaloacetate?

a) CO2

b) O2

c) Biotin

d) ATP

e) Pyruvate carboxylase

10) When insulin and AMP are low they allosterically affect PFK-1. Thus, when would

the synthesis of fructose 1,6-bisphosphatase be included?

a) A high carbohydrate meal

b) A high fat meal

c) A high protein meal

d) After a workout

e) While fasting

11) The conversion of glucose to glucose 6-phosphate (via gluctokinase) is induced by

____ and the conversion of glucose 6-phosphate to glucose (via glucose 6-phosphatase) is

induced by ____.

a) Insulin; Fasting

b) Fasting; Insulin

c) Insulin; Insulin

d) Fasting; Fasting

e) Insulin & Glucagon; Glucagon

12) The conversion of glucose to lactate yields how many ATPs? (Cori Cycle)

a) 0

b) 2

c) 4

d) 6

e) 8

13) The conversion of lactate to glucose requires how many ATPs? (Cori Cycle)

Page 67: Quiz Biochemistry

Biochemistry 14Mar2009

DO NOT DISTRIBUTE - 67 -

a) 0

b) 2

c) 4

d) 6

e) 8

14) The glycolytic enzyme glucokinase is induced by ____ and the gluconeogenic

enzyme phosphoenolpyruvate carboxykinase is repressed by ____.

a) Insulin; Glucagon

b) Glucagon; Insulin

c) Insulin; Insulin

d) Glucagon; Glucagon

15.1) After a high protein meal, insulin levels ____ and glucagon levels ____.

a) Rise drastically; Fall drastically

b) Fall drastically; Rise drastically

c) Rise drastically; Rise drastically

d) Fall drastically; Fall drastically

e) Rise slightly; Rise slightly

15.2) After a high glucose meal, insulin levels ____ and glucagon levels ____.

a) Rise drastically; Fall drastically

b) Fall drastically; Rise drastically

c) Rise drastically; Rise drastically

d) Fall drastically; Fall drastically

e) Rise slightly; Rise slightly

16) Glycogenolysis is ____ by glucagon and gluconeogenesis is ____ by glucagon.

a) Inactivated; Inactivated

b) Activated; Activated

c) Inactivated; Activated

d) Activated; Inactivated

17) What is the major source of blood fuel after fasting for 10 days?

a) Glucose

b) Free fatty acids

c) Glycerol

d) Ketone bodies

e) Lactate

32 – Digestion & Transportation of Dietary Lipids

1) How are the three fatty acids bonded to the glycerol backbone in a triacylglycerols?

a) Peptide bonded

b) Hydroxylation

c) Esterification

d) Hydrogenation

e) Dehydrogenation

2) Bile salts emulsify fats via detergent action. This requires bile salts to be:

a) Hydrophobic

b) Hydrophilic

c) Amphipathic

Page 68: Quiz Biochemistry

Biochemistry 14Mar2009

DO NOT DISTRIBUTE - 68 -

d) Cis double bonded

e) Trans double bonded

3) Pancreatic lipase works by increasing the pH in the intestinal lumen. It hydrolyzes

fatty acids of all chain lengths, producing free fatty acids and:

a) 1-monoacylglycerol (MG)

b) 2-monoacylglycerol (MG)

c) 3-monoacylglycerol (MG)

d) 4-monoacylglycerol (MG)

e) 6-monoacylglycerol (MG)

4.1) Bile salts must reach a particular concentration within the intestinal lumen before

they are effective agents for lipid digestion. This is because of which of the following?

a) The bile salt concentration must be equal to the triglyceride concentration

b) The bile salt solubility in the lumen is a critical factor

c) The ability of bile salts to bind lipase is concentration dependent

d) The bile salts cannot be reabsorbed in the ileum until they reach a certain

concentration

e) The bile salts do not activate lipase until they reach a particular concentration

4.2) What are apoproteins combined with to form chylomicrons?

a) 2-MG

b) Diacylglycerol

c) Triaclglycerol

d) Bile salts

e) Lipase colipase

4.3) What happens to the majority of bile salts when they reach the ileum?

a) Excreted from the body as waste

b) Returned to the pancreas

c) Returned to the stomach

d) Returned to the gallbladder

e) Returned to the liver

5.1) The conversion of nascent chylomicrons to mature chylomicrons requires which of

the following?

a) Bile salts

b) 2-MG

c) Lipoprotein lipase

d) High-density lipoprotein

e) Lymphatic system

5.2) During the life cycle of a chylomicrons, what activates LPL?

a) ApoC-II

b) ApoE

c) ApoB-48

d) ApoB-100

e) LDL

5.3) VLDL particles deliver newly synthesized TGs to muscle and adipose tissue via LPL

activity. If VLDL remnants acquire additional cholesterol from HDL particles in the

circulation, what can they become?

a) ApoC-II

Page 69: Quiz Biochemistry

Biochemistry 14Mar2009

DO NOT DISTRIBUTE - 69 -

b) ApoE

c) ApoB-48

d) ApoB-100

e) LDL

6.1) The most abundant component of chylomicrons is which of the following?

a) ApoB-48

b) Triglyceride

c) Phospholipid

d) Cholesterol

e) Cholesterol ester

6.2) The apoproteins B-48 and B-100 are similar with respect to which of the following?

a) They are synthesized from the same gene

b) They are derived by alternative splicing of the same hnRNA

c) ApoB-48 is a proteolytic product of apoB-100

d) Both are found in mature chylomicrons

e) Both are found in very-low-density lipoproteins

6.3) Type III hyperlipidemia is caused by a deficiency in apoprotein E. Analysis of the

serum of patient with this disorder would exhibit which of the following?

a) An absence of chylomicrons after eating

b) Above-normal levels of VLDL after eating

c) Normal triglyceride levels

d) Elevated triglyceride levels

e) Below-normal triglyceride levels

6.4) What is the major apoprotein associated with chylomicrons as they leave the

intestinal tract, serving as a major protein of the lipid carrier VLDL?

a) ApoC-II

b) ApoE

c) ApoB-48

d) ApoB-100

e) ApoA

33 – Synthesis of Fatty Acids, Triacylglycerols, Membrane Lipids

1.1) Which of the following is involved in the synthesis of triacylglycerols in adipose

tissue?

a) Fatty acids obtained from chylomicrons and VLDL

b) Glycerol 3-phosphate derived from blood glycerol

c) 2-MG as an obligatory intermediate

d) Lipoprotein lipase to catalyze the formation of ester bonds

e) Acetoacetyl CoA as an obligatory intermediate

1.2) Newly synthesized fatty acids are not immediately degraded because of which of the

following?

a) Tissues that synthesize fatty acids do not contain the enzymes that degrade

fatty acids

b) High NADPH levels inhibit !-oxidation

c) In the presence of insulin, the key fatty acid degrading enzyme is not induced

d) Newly synthesized fatty acids cannot be converted to their CoA derivatives

Page 70: Quiz Biochemistry

Biochemistry 14Mar2009

DO NOT DISTRIBUTE - 70 -

e) Transport of fatty acids into mitochondria is inhibited under conditions in

which fatty acids are being synthesized

1.3) In lipogenesis, citrate forms oxaloacetate and:

a) Palmitate

b) Malonyl CoA

c) Acetyl CoA

d) Glycerol-3-P

e) Apoproteins

2) NADPH for fatty acid synthesis comes from the pentose phosphate pathway

(glycolysis) and from what part of the TCA cycle?

a) Citrate synthase

b) Malic enzyme

c) Fumerase

d) Succinate dehydrogenase

e) Succinate thiokinase

3.1) A patient with hyperlipoproteinemia would be most likely to benefit from a low-

carbohydrate diet if the lipoproteins that are elevated in blood are which of the following?

a) Chylomicrons

b) VLDL

c) HDL

d) LDL

e) IDL

3.2) Fatty acids in the liver are converted to triacylglycerols, which are secreted into the

blood when packed with ____, allowing fatty acids to reach muscle and adipose.

a) Chylomicrons

b) VLDL

c) HDL

d) LDL

e) IDL

4.1) A molecule of palmitic acid, attached to carbon 1 of the glycerol moiety of a

triacylglycerol, is ingested and digested. It passes into the blood, is stored in a fat cell,

and ultimately is oxidized to carbon dioxide and water in a muscle cell. Choose the

molecular complex in the blood in which the palmitate residue is carried from the lumen

of the gut to the surface of the gut epithelial cell.

a) VLDL

b) Chylomicron

c) Fatty acid-albumin complex

d) Bile salt micelle

e) LDL

4.2) What is the rate-limiting enzyme of fatty acid synthesis?

a) Acetyl CoA carboxylase

b) Malic enzyme

c) Citrate lyase

d) Cytosolic malate dehydrogenase

e) Pyruvate dehydrogenase

Page 71: Quiz Biochemistry

Biochemistry 14Mar2009

DO NOT DISTRIBUTE - 71 -

4.3) A high insulin/glucagon ration would result in ____ of synthesis for acetyl CoA and

____ of synthesis for fatty acid synthase.

a) Inhibition; Induction

b) Induction; Inhibition

c) Induction; Induction

d) Inhibition; Inhibition

5) What fatty acid metabolism enzyme is active in its dephosphorylated state, with

dephosphorylation being catalyzed by an insulin-stimulated phosphatase.

a) Pyruvate dehydrogenase

b) Citrate lyase

c) Malic enzyme

d) Cytosolic malate dehydrogenase

e) Acetyl CoA carboxylase

6) Inhibition of carnitine:palmitoltransferase (CPTI) prevents newly synthesized fatty

acids from undergoing immediate oxiation by shuttling across the inner membrane of the

mitochondria. What inhibits CPTI?

a) FaCoA

b) Palmitate

c) FA synthase

d) Malonyl CoA

e) Acetyl CoA

7.1) How would a fasting individual with carnitine deficiency be affected?

a) Ability to oxidize long-chain fatty acids

b) Inability to oxidize short-chain fatty acids

c) Inability to produce acetyl CoA

d) Increased production of acetyl CoA

e) Difficulty forming ketone bodies

7.2) Which of the following would be seen in a patient with biotin deficiency?

a) Difficulty catabolizing even-chained fatty acids

b) Difficulty catabolizing odd-chained fatty acids

c) Over-catabolism of even-chained fatty acids

d) Over-catabolism of odd-chained fatty acids

7.3) A deficiency in pantothenic acid would affect which of the following?

a) CoA

b) Palmitate

c) Pyruvate

d) Glucose

e) Insulin

8.1) Fatty acid desaturation involves oxygen, NADH, and cytochrome b5, and occurs

where?

a) Nucleus

b) Lysosome

c) Peroxisome

d) Golgi

e) ER

Page 72: Quiz Biochemistry

Biochemistry 14Mar2009

DO NOT DISTRIBUTE - 72 -

8.2) What types of polyunsaturated fatty acids (named from the methyl end and related to

linoleic acid) are required for synthesis of eicosanoids but not synthesized de novo

(essential fatty acids)?

a) %2

b) %3

c) %4

d) %2 and %24

e) %3 and %6

9.1) Triacylglycerol synthesis starts in the liver with glycerol and in adipose tissue (and

liver) with glucose. What is the next intermediate formed by both of these starting points?

a) Phosphatidic acid

b) Diacylglycerol

c) Glycerol 3-P

d) Triacylglycerol

e) VLDL

9.2) What is formed from the phosphorylation of phosphatidic acid?

a) 2-MG

b) Diacyglycerol

c) Triacylglycerol

d) VLDL

e) HDL

9.3) VLDL is formed from cholesterol, phospholipids, and proteins that pack with

triacylglycerols, which is formed where?

a) SER

b) RER

c) Golgi

d) Nucleus

e) Mitochondria

9.4) What hormone stimulates the storage of triacylglycerols in adipose tissue?

a) Epinepherine

b) Norepinepherine

c) Adenosine

d) Insulin

e) Glucagon

10) What hormone, during fasting, causes cAMP levels to rise in adipose tissues and thus

stimulates lipolysis?

a) Epinepherine

b) Norepinepherine

c) Adenosine

d) Insulin

e) Glucagon

11.1) Which of the following is characteristic of sphingosine?

a) It is converted to ceramide by reacting with a UDP-sugar

b) It contains a glycerol moiety

c) It is synthesized from palmitoyl CoA and serine

d) It is a precursor of cardiolipin

Page 73: Quiz Biochemistry

Biochemistry 14Mar2009

DO NOT DISTRIBUTE - 73 -

e) It is only synthesized in neuronal cells

11.2) Which of the following would have a glycerol backbone, two fatty acid tails, and a

phosphate with head group attached?

a) Blood lipoproteins

b) Cardiolipin

c) Plasmalogens

d) Sphingomyelin

e) Gangliosides

11.3) What type of lipids are important in forming the myelin sheath surrounding nerves

in the central nervous system?

a) Triacyglycerols

b) Glycerophospholipids

c) Ether glycerolipids

d) Sphingophospholipids

e) Glycolipids

11.4) Along with sphingophospholipids, what type of lipid is a major component of

cellular membranes?

a) Triacyglycerols

b) Glycerophospholipids

c) Ether glycerolipids

d) Sphingolipids

e) Glycolipids

11.5) What type of lipids serve in intercellular communication and as the antigenic

determinants of the ABO blood group?

a) Sphingolipids

b) Ether glycerolipids

c) Glycolipids

d) Triacyglycerols

e) Glycerophospholipids

11.6) What type of lipids extend from the outer membrane of the cell and act as a

recognition site for specific chemicals?

a) Sphingolipids

b) Ether glycerolipids

c) Glycolipids

d) Triacyglycerols

e) Glycerophospholipids

12) What is the major component of lung surfactant?

a) Dipalmitoylphosphatidylcholine

b) Phosphatidylglycerol

c) Apoproteins

d) Cholesterol

e) Sp-A,B,C

34 – Cholesterol Absorption, Synthesis, Metabolism, and Fate

1) How are cholesterols absorbed from the intestinal lumen?

a) Diffusion

Page 74: Quiz Biochemistry

Biochemistry 14Mar2009

DO NOT DISTRIBUTE - 74 -

b) Primary active transport

c) Na+/K+ pump

d) Secondary active transport

e) Osmosis

2.1) Considering the final steps in cholesterol biosynthesis, when squalene is eventually

converted to lanosterol, which of the following statements is correct?

a) All of the sterols have three fused rings (the steroid nucleus) and are alcohols

with a hydroxyl group at C3

b) The action of squalene monooxygenase oxidizes carbon 14 of the squalene

chain, forming an epoxide

c) Squalene monooxygenase is considered a mixed function oxidase because it

catalyzes a reaction in which only one of the oxygen atoms of O2 is incorporated

into the organic substrate.

d) Squalene monooxygenase uses reduced flavin nucleotides (e.g. FADH2) as the

cosubstrate in the reaction

e) Squalene is joined at carbons 1 and 30 to form the fused ring structure of

sterols

2.2) What is the precursor for cholesterol?

a) Mevalonate

b) HMG-CoA

c) Acetoacetyl CoA

d) Acetyl CoA

e) Dimethylallyl pyrophosphate

2.3) Which of the following steps in the biosynthesis of cholesterol is the key regulator?

a) The condensation of acetoacetyl-CoA with a molecule of acetyl-CoA to yield

!-hydroxy-! methylglutaryl-CoA (HMC-CoA)

b) The reduction of HMG-CoA to mevalonate

c) The conversion of mevalonate to two activated isoprenes

d) The formation of farnesyl pyrophosphate

e) Condensation of six activated isoprene units to form squalene

3.1) Which of the following steps in the biosynthesis of cholesterol is the committed rate-

limiting step?

a) The condensation of acetoacetyl-CoA with a molecule of acetyl-CoA to yield

!-hydroxy-! methylglutaryl-CoA (HMC-CoA)

b) The reduction of HMG-CoA to mevalonate

c) The conversion of mevalonate to two activated isoprenes

d) The formation of farnesyl pyrophosphate

e) Condensation of six activated isoprene units to form squalene

3.2) What is the precursor for bile salts?

a) Mevalonate

b) HMG-CoA

c) Acetoacetyl CoA

d) Acetyl CoA

e) Cholesterol

4) Cholesterol esters are packed into VLDLs and secreted from the hepatocyte into the

blood to reach tissues that require greater amounts of cholesterol than can be synthesized

Page 75: Quiz Biochemistry

Biochemistry 14Mar2009

DO NOT DISTRIBUTE - 75 -

de novo. They help in the formation of steroid hormones, cellular membranes, and what

vitamin?

a) A

b) D

c) E

d) K

e) C

5) Bacteria in the gut (colonic anerobes) ____ and ____ bile salts, allowing them to be

reabsorbed and remain in the enterohepatic circulation.

a) Conjugate; Hydroxylate

b) Conjugate; Dehydroxylate

c) Deconjugate; Hydroxylate

d) Deconjugate; Dehydroxylate

6.1) Which one of the following apoproteins acts as a cofactor activator of the enzyme

lipoprotein lipase (LPL)?

a) ApoC-III

b) ApoC-II

c) ApoB-100

d) ApoB-48

e) ApoE

6.2) Which one of the following sequences places the lipoproteins in the order of most

dense to least dense?

a) HDL > VLDL > chylomicrons > LDL

b) HDL > LDL > VLDL > chylomicrons

c) LDL > chylomicrons > HDL > VLDL

d) VLDL > chylomicrons > LDL > HDL

e) LDL > chylomicrons > VLDL > HDL

6.3) Which of the following delivers endogenous lipids to the liver and is a precursor to

LDL?

a) Chylomicrons

b) Chylomicron remnants

c) VLDL

d) IDL

e) HDL

6.4) What is the role of chylomicrons remnants?

a) Deliver dietary lipids

b) Return endogenous lipids to the liver

c) Return dietary lipids to the liver

d) Deliver endogenous lipids

e) Reverse cholesterol transport

7.1) Which of the following has a receptor on the plasma membrane containing clatherin

for receptor-mediated endocytosis and conversion to cholesterol?

a) Chylomicrons

b) LDL

c) VLDL

d) IDL

Page 76: Quiz Biochemistry

Biochemistry 14Mar2009

DO NOT DISTRIBUTE - 76 -

e) HDL

7.2) How many major regions are seen in the structure of the LDL receptor?

a) 2

b) 3

c) 4

d) 5

e) 6

8.1) A patient is diagnosed with familial hypercholesterolemia (FH) type IIA, which is

caused by genetic defects in the gene that encodes the LDL receptor. After testing, there

is an elevation in which of the following in the blood?

a) TG

b) Cholesterol

c) PL

8.2) A patient with excess chylomicrons would have an elevation in which of the

following in the blood?

a) TG

b) Cholesterol

c) PL

9) Where do foam cells form during atherosclerosis to deform the vessel inward (lumen)

and increase the likelihood of a clot?

a) Tunica adventitia

b) Tunica media

c) Internal elastic lamina

d) Subintimal space

e) Endothelial cell

10.1) Of the major risk factors for the development of atherosclerotic cardiovascular

disease (ASCVD) such as sedentary lifestyle, obesity, cigarette smoking, diabetes

mellitus, hypertension, and hyperlipidemia, which one, if present, is the only risk factor

in a given patient without a history of having had a myocardial infarction that requires

that the therapeutic goal for the serum LDL cholesterol level be < 100mg/dL?

a) Obesity

b) Cigarette smoking

c) Diabetes mellitus

d) Hypertension

e) Sedentary lifestyle

10.2) It has been reported that high concentrations of which of the following correlate

with an increased risk of coronary artery disease, even in patient in whom the lipid profile

is otherwise normal, as they cannot be converted to active plasmin?

a) HMG-CoA

b) HDL

c) VLDL

d) Lp(a)

e) Chylomicrons

11) Cholesterol, a precursor to many steroid hormones, allows them to:

a) Dissolve in blood plasma

b) Cross the blood-brain barrier

Page 77: Quiz Biochemistry

Biochemistry 14Mar2009

DO NOT DISTRIBUTE - 77 -

c) Act as an extracellular signal

d) Use cytochrome P450

e) Cross plasma membranes

12) What produces cholecalciferol in the skin, hydroxylating 7-dehydrocholesterol to

form active vitamin D?

a) Insulin

b) Glucagon

c) Free radicals

d) UV light

e) Lipid synthesis

35 – Metabolism of Eicosanoids

1.1) What eicosanoid metabolic enzyme is used in the conversion of arachidonic acid to

prostaglandins and thromboxanes?

a) Lipoxygenase

b) Cyclooxygenase

c) Phospholipase A2

d) Diacylglycerol lipase

e) Cytochrome P450

1.2) In humans, prostaglandins are primarily derived from which of the following?

a) Glucose

b) Acetyl CoA

c) Arachidonic acid

d) Oleic acid

e) Leukotrienes

1.3) What eicosanoid metabolic enzyme in involved in the conversion of arachidonic acid

to leukotrienes?

a) Lipoxygenase

b) Cyclooxygenase

c) Phospholipase A2

d) Diacylglycerol lipase

e) Cytochrome P450

1.4) What eicosanoid metabolic reaction involves cytochrome P450?

a) Arachidonic acid to membrane phospholipid

b) Membrane phospholipid to arachidonic acid

c) Arachidonic acid to epoxides

d) Arachidonic acid to prostaglandins

e) Arachidonic acid to leukotrienes

1.5) What eicosanoid metabolic reaction involves the enzyme phospholipase A2?

a) Arachidonic acid to membrane phospholipid

b) Arachidonic acid to leukotrienes

c) Arachidonic acid to epoxides

d) Arachidonic acid to prostaglandins

e) Membrane phospholipid to arachidonic acid

2.1) Aspirin will inhibit which of the following reaction pathways?

a) Arachidonic acid to thromboxanes

Page 78: Quiz Biochemistry

Biochemistry 14Mar2009

DO NOT DISTRIBUTE - 78 -

b) Arachidonic acid to leukotrienes

c) Arachidonic acid to phospholipids

d) Linoleic to arachidonic acid

e) Acetyl CoA to linoleic acid

2.2) Which of the following drugs leads to the covalent modification, and inactivation

(blocking), of both COX-1 and COX-2 enzymes?

a) Aspirin

b) Tylenol

c) Celebrex

d) Vioxx

e) Advil

2.3) Glucocorticoids will inhibit which of the following reaction enzymes?

a) Cytochrome P450

b) Cyclooxygenase

c) Diacylglycerol lipase

d) Phospholipase A2

e) Lipoxygenase

3.1) Alpha naming of phosphoglandins is based on their ring configuration and

substituents. An E-series phosphoglandin (ring with OH and double bonded O) has

double bonds between carbons 13 and 14 (trans), 5 and 6 (cis), and 17 and 18 (cis). How

would this be named?

a) PGE1

b) PGE2

c) PGE3

d) PGE4

e) PGE6

3.2) How would an F-series phosphoglandin be named if the hydroxyl group at carbon 9

primarily existed in the " position and there were two unsaturated bonds along the linear

portion of the hydrocarbon chain?

a) PGF2

b) PGF"

c) PGF4!

d) PGF2"

e) PGF2!

4) What prostaglandin series do fish oils and other omega-3 fatty acids give rise to?

a) Series 1

b) Series 2

c) Series 3

d) Series 2"

e) Series 2!

5.1) Thromboxane A2, which is found in high levels in platelets, aids in wound repair

through induction of which of the following activities?

a) Inhibits COX-2 gene expression

b) Inhibits COX-1 gene expression

c) Vasoconstriction

d) Vasodilation

Page 79: Quiz Biochemistry

Biochemistry 14Mar2009

DO NOT DISTRIBUTE - 79 -

e) Bronchodilation

5.2) Certain prostaglandins, when binding to their receptor, induce an increase in

intracellular calcium levels. The signal that leads to the elevation of intracellular calcium

is initiated by which of the following enzymes?

a) Protein kinase A

b) Phospholipase C

c) Phospholipase A2

d) Protein kinase C

e) Cyclooxygenase

5.3) Which of the following would decrease platelet aggregation, increase cAMP, and

cause vasodilation?

a) Prostaglandins

b) Leukotrienes

c) Thromboxanes

5.4) Which of the following is NOT a function of leukotrienes?

a) Increase vascular permeability

b) Increase T-cell proliferation

c) Increase leukocyte aggregation

d) Bronchoconstriction

e) Vascoconstriction

36 – Integration of Carbohydrate and Lipid Metabolism

1.1) A 20-year-old woman with diabetes mellitus was admitted to the hospital in a

semiconscious state with fever, nausea, and vomiting. Her breath smelled of acetone. A

urine sample was strongly positive for ketone bodies. Which of the following statements

is correct concerning this woman?

a) A blood glucose test would probably show that her blood glucose level was

well below 80 mg/dL

b) An insulin injection would decrease her ketone body production

c) She should be given a glucose infusion to regain consciousness

d) Glucagon should be administered to stimulate glycogenolysis and

gluconeogenesis in the liver

e) The acetone was produced by decarboxylation of ketone body !-

hydroxybutyrate

1.2) A woman was told by her physician to go on a low-fat diet. She decided to continue

to consume the same number of calories by increasing her carbohydrate intake while

decreasing her fat intake. Which of the following blood lipoprotein levels would be

decreased as a consequence of her diet?

a) VLDL

b) IDL

c) HDL

d) Chylomicrons

e) HDL

1.3) A chronic alcoholic has been admitted to the hospital because of a severe

hypoglycemic episode brought about by excessive alcohol consumption for the past 5

Page 80: Quiz Biochemistry

Biochemistry 14Mar2009

DO NOT DISTRIBUTE - 80 -

days. A blood lipid analysis indicates much higher than expected VLDL levels. The

elevated VLDL is attributable to which of the following underlying causes?

a) Alcohol-induced inhibition of lipoprotein lipase

b) Elevated NADH levels in the liver

c) Alcohol-induced transcription of the apoB-100 gene

d) NADH activation of phosphoenolpyruvate carboxykinase

e) Acetaldehyde induction of enzymes on the endoplasmic reticulum

1.4) Certain patients with abetalipoproteinemia frequently have difficulties in maintaining

blood volume; their blood has trouble clotting. This symptom is attributable to which of

the following?

a) Inability to produce chylomicrons

b) Inability to produce VLDL

c) Inability to synthesize clotting factors

d) Inability to synthesize fatty acids

e) Inability to absorb short-chain fatty acids

1.5) Which of the following is NOT true regarding the effects of insulin dependent

diabetes mellitus (IDDM)? (decreased insulin levels)

a) Lipolysis is increased leading to fatty acid production

b) Number of available GLUT 4 receptors is decreased

c) HMG-CoA synthase activity leads to high production of ketone bodies

d) VLDL synthesis is increased raising serum VLDL concentrations

e) Lipoprotein lipase production is increased

2) When glucagon (or epinephrine) binds to its cell membrane receptor, initially

activating a G protein, levels of which of the following rise?

a) ATP

b) cAMP

c) cGMP

d) G6P

e) F6P

3.1) Assume that an individual has been eating excess calories daily such that they will

gain weight. Under which of the following conditions would the person gain weight most

rapidly?

a) If all the excess calories were due to carbohydrate

b) If all the excess calories were due to triacylglycerol

c) If all the excess calories were split 50%-50% between carbohydrate and

triacylglycerol

d) If all the excess calories were split 25%-75% between carbohydrate and

triacylglycerol

e) It makes no difference what form the excess calories are in

3.2) Insulin ____ the secretion of lipoprotein lipase (LPL) from adipose tissues and ____

the transport of glucose into these cells.

a) Stimulates; Inhibits

b) Inhibits; Stimulates

c) Stimulates; Stimulates

d) Inhibits; Inhibits

4) What enzyme activity is NOT increased during gluconeogenesis in the liver?

Page 81: Quiz Biochemistry

Biochemistry 14Mar2009

DO NOT DISTRIBUTE - 81 -

a) Phosphoenolpyruvate carboxykinase

b) Fructose 1,6-bisphosphatase

c) Glucose 6-phosphate

d) Glucokinase

37 – Protein Digestion and Amino Acid Absorption

1) Which of the following enzymes is NOT involved in protein digestion?

a) Pepsin

b) Trypsin

c) Chymotrypsin

d) Amylase

e) Elastase

2) Which of the following secretes bicarbonate to raise the pH of the digestive tract and

activate proteases?

a) Liver

b) Gallbladder

c) Pancreas

d) Stomach

e) Duodenum

3.1) An individual with a deficiency in the conversion of trypsinogen to trypsin would be

expected to experience a more detrimental effect on protein digestion than an individual

who was defective in any of the other digestive proteases. This is due to which of the

following?

a) Trypsin has a greater and wider range of substrates to act on

b) Trypsin activates pepsinogen, so digestion can begin in the stomach

c) Trypsin activates the other zymogens that are secreted by the pancreas

d) Trypsin activates enteropeptidase, which is needed to activate the other

pancreatic zymogens

e) Trypsin inhibits intestinal motility, so the substrates can be hydrolyzed for

longer periods

3.2) Which of the following enzymes is activated through an autocatalytic process?

a) Enteropeptidase

b) Trypsinogen

c) Pepsinogen

d) Aminopeptidase

e) Proelastase

3.3) Enteropepsidase converts which of the following zymogens to its active form?

a) Pepsinogen

b) Trypsinogen

c) Chymotripsinogen

d) Proelastase

e) Procarboxypeptidase

3.4) Which of the following zymogens does NOT use trypsin to activate it?

a) Pepsinogen

b) Procarboxypeptidase

c) Chymotripsinogen

Page 82: Quiz Biochemistry

Biochemistry 14Mar2009

DO NOT DISTRIBUTE - 82 -

d) Proelastase

4.1) Kwashiorkor can result from which of the following?

a) Consuming a calorie-deficient diet that is also deficient in protein

b) Consuming a calorie-adequate diet that is deficient in carbohydrates

c) Consuming a calorie-adequate diet that is deficient in fatty acids

d) Consuming a calorie-adequate diet that is deficient in proteins

e) Consuming a calorie-deficient diet that is primarily proteins

4.2) Children with kwashiorkor usually have a fatty liver. This is due to which of the

following?

a) The high fat content of their diet

b) The high carbohydrate content of their diet

c) The high protein content of their diet

d) The lack of substrates for gluconeogenesis in the liver

e) The lack of substrates for protein synthesis in the liver

f) The lack of substrates for glycogen synthesis in the liver

4.3) Which of the following is a result of cystic fibrosis, which is a defect in the function

of chloride channels?

a) Inability to produce pancreatic enzymes

b) Inability to activate pancreatic zymogens

c) Inability to secrete pancreatic enzymes

d) Inability to digest starches

e) Inability to digest fatty acids

4.4) What would be the likely result if the secretory trypsin inhibitor was not present?

a) Hepatitis due to activated zymogens

b) Pancreatitis due to autodigestion

c) Appendicitis

d) Cholecystitis due to blockage of bile duct

e) Splenic inflammation due to lymph production

5) What type of transport is the transepithelial amino acid system, which involves Na+

dependent carriers?

a) Facilitated Diffusion

b) Simple diffusion

c) Primary active transport

d) Secondary active transport

e) Voltage-gated ion channels

6) Cystinuria and Hartnup disease involve defects in two different transport proteins.

What would be seen with these diseases relative to the intestinal and renal epithelial

cells?

a) Poor absorption of triacylglycerides

b) Poor absorption of carbohydrates

c) Poor absorption of amino acids

d) Complete absorption of proteins

e) Complete absorption of amino acids

7.1) What family of proteases is involved in lysosomal protein turnover?

a) Cathepsin

b) Calpains

Page 83: Quiz Biochemistry

Biochemistry 14Mar2009

DO NOT DISTRIBUTE - 83 -

c) Caspases

d) Proteasome

e) Serine proteases

7.2) The three-enzyme system that adds ubiqutin to proteins targets these proteins for:

a) Nuclear storage

b) Epimerization

c) Cytosolic storage

d) Degradation

e) Secretion from the cell

8) Cysteinuria is a disorder where cystine and basic amino acids are not absorbed and

thus are concentrated in urine, leading to kidney stones. Since these are non-essential

amino acids, they do not show enzyme deficiencies. Which of the following is NOT

involved in cysteinuria?

a) Cystine

b) Lysine

c) Arginine

d) Ornithine

e) Tryptophan

9) The #-glutamyl cycle is necessary for synthesis of glutathione, which protects cells

from:

a) Oxidative damage

b) Reductive damage

c) Hydrolysis

d) Lysosomal storage disorders

e) Auto digestion

38 – Fate of Amino Acid Nitrogen: Urea Cycle

1.1) The major regulated step of the urea cycle is which of the following?

a) Carbamoyl phosphate synthetase I

b) Ornithine transcarbamoylase

c) Argininosuccinate synthetase

d) Argininosuccinate lyase

e) Arginase

1.2) What are the two direct sources of nitrogen (amino acids) in the creation of urea?

a) Aspartate and NH3

b) Glutamate and NH3

c) Aspartate and NH4+

d) Glutamate and NH4+

2.1) The nitrogens in urea are directly derived from which of the following compounds?

a) Ornithine and carbamoyl phosphate

b) Ornithine and aspartate

c) Ornithine and glutamate

d) Carbamoyl phosphate and aspartate

e) Carbamoyl phosphate and glutamine

f) Aspartate and glutamine

Page 84: Quiz Biochemistry

Biochemistry 14Mar2009

DO NOT DISTRIBUTE - 84 -

2.2) In the fasting state, glutamine in the kidney yields NH3 and excretes NH4+ into the

urine. What amino acid is created as well that transports to the liver?

a) Alanine

b) Arginine

c) Asparagine

d) Glycine

e) Valine

3) Which of the following is NOT a biochemical reaction for removal of nitrogen from

amino acids?

a) Glutamate to "-ketoglutarate

b) Histidine to urocanate

c) Serine to pyruvate

d) Glutamine to asparagine

e) Threonine to "-ketobutyrate

4) Pyridoxal phosphate, which is required for transaminations, is also required for which

of the following pathways?

a) Glycolysis

b) Gluconeogenesis

c) Glycogenolysis

d) TCA cycle

e) Fatty acid oxidation

5) Which of the following is NOT a symptom of PLP (Vitamin B6) deficiency?

a) Dermatitis

b) Hypothyroidism

c) Microcytic hypochromic anemia

d) Weakness

e) Irritability

6) Which of the following is NOT a reaction that produces ammonia in the body?

a) Bacteria in the lumen of the GI tract react to form ammonia

b) Deamination of certain amino acids

c) The purine nucleotide cycle in muscle

d) The purine nucleotide cycle in the brain

e) The purine nucleotide cycle in the liver

7.1) Which one of the following enzymes can fix ammonia into an organic molecule?

a) Alanine-pyruvate aminotransferase

b) Glutaminase

c) Glutamate dehydrogenase

d) Arginase

e) Argininosuccinate synthetase

7.2) Why is ammonia absorbed by the intestine not immediately toxic?

a) It participates in the glutamine to glutamate reaction

b) It enters the purine nucleotide cycle

c) It participates in the asparagines to aspartate reaction

d) It is broken down by bacteria to urea

e) The intestinal wall is not permeable to ammonia ions

8) What form does nitrogen take when excreted by the kidney cells?

Page 85: Quiz Biochemistry

Biochemistry 14Mar2009

DO NOT DISTRIBUTE - 85 -

a) Urine

b) Glutamate

c) Fumerate

d) RBCs

e) Lymph

9) Ammonium ion (NH4+) can be converted into ammonia (NH3) at a pK of 9.3. This is

important in reducing the acidity of urine. At around physiological pH, which of the

following is true?

a) NH4+ is 10 times more concentrated than NH3

b) NH4+ is 100 times more concentrated than NH3

c) NH3 is 10 times more concentrated than NH4+

d) NH3 is 100 times more concentrated than NH4+

e) NH3 is 1000 times more concentrated than NH4+

10) Why are blood levels of alanine and glutamine much higher than concentrations of

other amino acids?

a) They donate ammonia most readily

b) They form "-ketobutyrate

c) They form "-ketogluterate

d) They carry nitrogen to the liver

e) They form urea in the GI tract

11) Which of the following, which coordinately funnel amino groups into urea, is

particularly important to the kidney where NH4+ is excreted directly into the urine?

a) Aminotransferase

b) Glutamate dehydrogenase

c) Glutaminase

12) During exercise when the muscle uses blood-borne glucose, what amino acid is

pyruvate converted to with the help of glutamate and "-ketogluterate?

a) Valine

b) Arginine

c) Glycine

d) Asparagine

e) Alanine

13) With the addition of NH4+ and the use of NADPH, which of the following reactions

would use glutamate dehydrogenase in steps to rid the body of toxic ammonia to urine?

a) "-ketogluterate to Glutamate

b) "-ketogluterate to Glutamine

c) "-ketogluterate to Urea

d) Glutamate to "-ketogluterate

e) Glutamine to "-ketogluterate

14.1) In what forms does nitrogen enter the urea cycle?

a) Aspartate and NH3

b) Glutamate and NH3

c) Aspartate and NH4+

d) Glutamate and NH4+

e) Bicarbonate and NH4+

14.2) Which of the following is NOT an intermediate of the urea cycle?

Page 86: Quiz Biochemistry

Biochemistry 14Mar2009

DO NOT DISTRIBUTE - 86 -

a) Citrulline

b) Alanine

c) Argininosuccinate

d) Carbomoyl phosphate

e) Ornithine

14.3) What is the role of n-acetylglutamate (NAG) in the urea cycle?

a) Activate glutamate

b) Activate acetyl CoA

c) Activate arginine

d) Activate CPSI

e) Activate ornithine transcarbamoylase

15) Which of the following enzymes if found in the blood would NOT indicate liver

damage (as they leaked from the liver to the blood)?

a) Total bilirubin

b) ALT

c) AST

d) Alkaline phosphatase

e) CPK

16) Neural toxicity of NH4+ is due to depletion of TCA cycle intermediates and which of

the following in the central nervous system?

a) Ca++

b) ATP

c) cGMP

d) Calmodulin

e) Acetyl CoA

17) When would nitrogen secretion in urea be the highest?

a) After a meal high in carbohydrates

b) After a meal high in triacylglycerides

c) After a mixed meal, 30 minutes after exercising

d) 12 hours into the fasting state

e) 6 weeks into the fasting state

Match the following lab results with the defective enzyme. All cases have low BUN:

a) Carbamolyphosphate synthetase I

b) Ornithine transcarbamoylase

c) Argininosuccinate synthase

d) Argininosuccinate lyase

e) Arginase

Urine orotate Blood citrulline Blood arginine Blood ammonia

18.1) Low Low Low High

18.2) Unknown Very High Low High

18.3) - - High Moderately High

18.4) High Low Low High

18.5) - High Low High

19) Which of the following would NOT be a treatment option for disorders of ammonia

removal and urea synthesis?

a) High protein diet

Page 87: Quiz Biochemistry

Biochemistry 14Mar2009

DO NOT DISTRIBUTE - 87 -

b) Massive arginine supplementation

c) Benzoid acid

d) Phenylbutyrate

e) Gene therapy

39 – Synthesis and Degradation of Amino Acids

1.1) Which of the following nutritionally essential amino acids are required for growth,

especially in children?

a) Arginine and histidine

b) Tryptophan and lysine

c) Methionine and phenylalanine

d) Threonine and valine

e) Leucine and isoleucine

1.2) Which of the following nutritionally essential amino acids are considered solely

ketogenic (not glucogenic)?

a) Arginine and valine

b) Tryptophan and isoleucine

c) Methionine and phenylalanine

d) Threonine and histidine

e) Leucine and lysine

2.1) The degradation of amino acids can be classified into families, which are named

after the end product of the degradation pathway. Which of the following is such an end

product?

a) Citrate

b) Glyceraldehyde-3-phosphate

c) Fructose-6-phosphate

d) Malate

e) Succinyl-CoA

2.2) Which of the following amino acids is NOT associated with cofactors propionyl

CoA, methylmalonyl CoA, and succinyl CoA.

a) Valine

b) Threonine

c) Aspartate

d) Isoleucine

e) Methionine

2.3) What cofactor is directly needed for leucine metabolism?

a) Propionyl CoA

b) Methylmalonyl CoA

c) Succinyl CoA

d) HMG CoA

e) Acetyl CoA

3.1) A folic acid deficiency would interfere with the synthesis of which of the following

amino acids from the indicated precursors?

a) Aspartate from oxaloacetate and glutamate

b) Glutamate from glucose and ammonia

c) Glycine from glucose and alanine

Page 88: Quiz Biochemistry

Biochemistry 14Mar2009

DO NOT DISTRIBUTE - 88 -

d) Proline from glutamate

e) Serine from glucose and alanine

3.2) What amino acid do certain tumor cells need to grow, thus making the amino acid

enzyme an anti-tumor agent?

a) Arginine

b) Asparagine

c) Methionine

d) Threonine

e) Leucine

3.3) Which of the following amino acid degradation pathway disorders would lead to

renal failure due to stone formation?

a) Phenylalanine

b) Tyrosine

c) Methionine

d) Glycine

e) Branched-chain amino acids

4) Which of the following is NOT an amino acid that can be made from and/or

metabolized back to glycolytic intermediates?

a) Threonine

b) Alanine

c) Cysteine

d) Glycine

e) Serine

5) Which of the following is NOT an amino acid that can be made from, or metabolized

to, citric acid cycle intermediates?

a) Proline

b) Aspartate

c) Arginine

d) Histidine

e) Glutamate

6) Which amino acid is considered essential in children because they cannot generate an

adequate amount via the urea cycle?

a) Histidine

b) Glutamine

c) Arginine

d) Proline

e) Glutamate

7) Homocystinuria is caused by deficiencies in enzymes that are responsible for

metabolism of what amino acid?

a) Phenylalanine

b) Tyrosine

c) Methionine

d) Glycine

e) Alanine

8.1) If an individual has a vitamin B6 deficiency, which of the following amino acids

could still be synthesized and be considered nonessential?

Page 89: Quiz Biochemistry

Biochemistry 14Mar2009

DO NOT DISTRIBUTE - 89 -

a) Tyrosine

b) Serine

c) Alanine

d) Cystein

e) Aspartate

8.2) Homocystinuria and macrocytic anemia can be caused by deficiencies in which

vitamins?

a) B12 and B6 (pyridoxine)

b) B12 and B7 (biotin)

c) B7 and B9 (folate)

d) A and B3 (niacin)

e) B12 and B9 (folate)

9) What type of amino acids accumulate and are excreted into the urine giving a burnt

sugar odor in maple syrup urine disease?

a) Acidic

b) Basic

c) Neutral

d) Branched-chain

e) Aromatic

10) A newborn infant has elevated levels of phenylalanine and phenylpyruvate in her

blood. Which of the following enzymes might be deficient in this baby?

a) Phenylalanine dehydrogenase

b) Phenylalanine oxidase

c) Dihydropteridine reductase

d) Tyrosine hydroxylase

e) Tetrahydrofolate synthase

11) Ketogenic amino acids have a carbon skeleton that is catabolized to acetyl CoA or

acetoacetate, which can enter the pathway of ketone body synthesis in the liver. Which of

the following is NOT a set of amino acids that are considered ketogenic?

a) Phenylalanine and tyrosine

b) Methionine and valine

c) Threonine and lysine

d) Leucine and isoleucine

e) Tryptophan

40 – Tetrahydrofolate, Vitamin B12 , and S-Adenosylmethionine

1.1) Propionic acid accumulation from amino acid degradation would result from a

deficiency of which of the following vitamins?

a) Vitamin B6

b) Biotin

c) Folic acid

d) Vitamin B12

e) Vitamin B1

f) Vitamin B2

1.2) A dietary vitamin B12 deficiency can result from which of the following?

a) Excess intrinsic factor production by the gastric parietal cells

Page 90: Quiz Biochemistry

Biochemistry 14Mar2009

DO NOT DISTRIBUTE - 90 -

b) Eating a diet high in animal protein

c) Pancreatic insufficiency

d) Increased absorption of folic acid

e) Inability to conjugate the vitamin with glutamic acid

2.1) Which of the following forms of tetrahydrofolate is required for the synthesis of

methionine from homocysteine?

a) N5, N10-methylene tetrahydrofolate

b) N5-methyl tetrahydrofolate

c) N5, N10-methenyl tetrahydrofolate

d) N10-formyl tetrahydrofolate

e) N5-formimino tetrahydrofolate

2.2) Which of the following is the form of folate found in the diet?

a) FH4

b) N10-formyl FH4

c) N5, N10-methenyl FH4

d) N5, N10-methylene FH4

e) N5-methyl FH4

3) Biochemically, why is folate needed?

a) Cellular division

b) Production of ATP

c) Production of NADPH

d) Oxidation of free radicals

e) Absorption of essential amino acids

4) What blocks the conversion of dUMP to dTMP via FH4, inhibiting DNA purine

synthesis?

a) Serine

b) Glycine

c) 2-MP

d) 5-FU

e) Mannose-6-P

5) After B12 is freed in the intestine by pancreatic proteases, what does it bind to?

a) R-binders

b) Intrinsic factors

c) Transcobalamins

d) Parietal cells

e) Gastric mucosa

6) Pernicious anemia is caused by the inability to absorb B12 due to lack of which of the

following?

a) R-binders

b) Intrinsic factors

c) Transcobalamins

d) Parietal cells

e) Normal gastric mucosa

Match the following precursor with the produce methylated via S-adenosylmethionine:

7.1) Norepinepherine a) Melatonin

7.2) Acetyleserotonin b) Phosphatidylcholine

Page 91: Quiz Biochemistry

Biochemistry 14Mar2009

DO NOT DISTRIBUTE - 91 -

7.3) Phosphatidylethanolamine c) Methylated nucleotides

7.4) Guanidinoacetate d) Creatine

7.5) Nucleotides e) Epinepherine

8) Which form of FH4 requires B12 to release the methyl group, resulting in a methyl-

trap and the inability to remove carbons from folate?

a) FH4

b) N10-formyl FH4

c) N5, N10-methenyl FH4

d) N5, N10-methylene FH4

e) N5-methyl FH4

9) Folate replacement therapy can bypass the methyl-trap and thus correct the

hematopoetic defect in a vitamin B12-deficient patient. However, what defect would

NOT be corrected unless B12 was given as well?

a) Cardiac

b) Renal

c) Hepatic

d) Neural

e) Muscular

10.1) An alternative method to methylate hemocysteine to form methionine is which of

the following?

a) Using glycine and FH4 as the methyl donor

b) Using dimethylglycine as the methyl donor

c) Using choline as the methyl donor

d) Using sarcosine as the methyl donor

e) Using betaine as the methyl donor

10.2) Hyperhomocysteinemia is a disease where homocysteine levels can accumulate via

both folic acid and vitamin B12. What diseases are linked to this disorder?

a) Neurological and renal diseases

b) Renal and hepatic diseases

c) Muscular and peripheral nervous system diseases

d) Cardiac and neurological diseases

e) Splenic and cardiac

Match the following deficiency/clinical problem with the patient of concern:

11.1) Folate deficiency a) Vegan

11.2) B12 and folate deficiency b) Alcoholic

11.3) B12 deficiency and methyl-trapping of THF c) Pregnant

12) What clinical issue is seen in newborns that had folate-deficient mothers?

a) Cleft lip/palate

b) Cardiac defects

c) Neural tube defect

d) Oxycephaly

e) Cyclopia

13) Which of the following builds up in myelin sheaths, along with methylmalonic acid,

seen in vitamin B12 deficiency?

a) Arachidonic acids

b) Even-chain fatty acids

Page 92: Quiz Biochemistry

Biochemistry 14Mar2009

DO NOT DISTRIBUTE - 92 -

c) Odd-chain fatty acids

d) Polar amino acids

e) Non-polar amino acids

14) Which of the following is seen in vitamin B12 deficiency but not in folate

deficiency?

a) Homocystinemia

b) Megaloblastic anemia

c) Increased risk of cardiovascular problems

d) Associated with severe malnutrition

e) Leads to peripheral neuropathy

41 – Purine and Pyrimidine Synthesis

1) Which of the following is NOT a disorder associated with purine and/or pyrimidine

metabolism?

a) Hereditary orotic aciduria

b) Porphyria

c) Lesch-Nyhan syndrome

d) Adenosine deaminase deficiency (SCID)

e) Purine nucleoside phosphorylase deficiency

2) Which of the following is NOT a function of nucleotides?

a) DNA and RNA precursors

b) Metabolic allosteric regulators

c) Decrease blood glucose

d) Form structural moieties on coenzymes

e) Work in energy metabolism

3) What are the two major sites of de novo synthesis of purines?

a) Liver and kidney

b) Liver and intestines

c) Liver and gallbladder

d) Liver and pancreas

e) Liver and brain

4) Which of the following folates provides a carbon twice in the reaction to IMP for

purine synthesis?

a) FH4

b) N10-formyl FH4

c) N5, N10-methenyl FH4

d) N5, N10-methylene FH4

e) N5-methyl FH4

5) Which of the following cells uses salvage of free bases as its major form of nucleotide

generation?

a) Lymphocytes

b) RBCs

c) Neurons

d) Osteoclasts

e) Nephron cells

Page 93: Quiz Biochemistry

Biochemistry 14Mar2009

DO NOT DISTRIBUTE - 93 -

6) Which of the following cell types is most affected in patients with a deficiency in

purine nucleoside phosphorylase activity?

a) RBCs

b) Eosinophils

c) B-cells

d) T-cells

e) Natural killer cells

7) Which of the following folates forms dTMP with glutamine for pyrimidine synthesis?

a) FH4

b) N10-formyl FH4

c) N5, N10-methenyl FH4

d) N5, N10-methylene FH4

e) N5-methyl FH4

8) How are nucleotide bases transported through the body to other cellular destinations?

a) Via neurons

b) Via lymph vessels

c) Via RBCs

d) Via white blood cells

e) Via lipids in plasma

9.1) What are the precursors of purine production?

a) PRPP and glutamine

b) PRPP and glycine

c) PRPP and aspartate

d) RSP and glutamine

e) RSP and glycine

9.2) In pyrimidine production, UTP ____ and PRPP ____ activity of the initial step from

glutamine + CO2 + 2ATP to carbamoyl phosphate.

a) Increases; Increases

b) Decreases; Decreases

c) Increases; Decreases

d) Decreases; Increases

10) Hereditary orotic aciduria involves a defect in the UMP synthase gene, resulting in

inability to synthesize pyrimidines. Which of the following is a major clinical symptom

of this disease?

a) Painful joints

b) Loss of immune system

c) Partial loss of immune system

d) Mental retardation and self-mutilation

e) Growth retardation

11) Which of the following must happen to the ribose moiety for DNA synthesis?

a) It must be oxidized

b) It must be reduced

c) It must be polymerized

d) It must be catabolized

e) It must be stabilized

Page 94: Quiz Biochemistry

Biochemistry 14Mar2009

DO NOT DISTRIBUTE - 94 -

12.1) Gout (overload of uric acid in the body) can result from a reduction in activity of

which one of the following enzymes?

a) Glutamine phosphoribosyl amidotransferase

b) Glucose 6-phosphatase

c) Glucose 6-phosphate dehydrogenase

d) PRPP synthetase

e) Purine nucleoside phosphorylase

12.2) Allopurinol can be used to treat gout because of its ability to inhibit which one of

the following reactions?

a) AMP to XMP

b) Xanthine to uric acid

c) Inosine to hypoxanthine

d) IMP to XMP

e) XMP to GMP

12.3) Patients with gout should avoid eating food containing purines as it increases uric

acid formation. Which of the following is a major clinical symptom of gout?

a) Painful joints

b) Loss of immune system

c) Partial loss of immune system

d) Mental retardation and self-mutilation

e) Growth retardation

13.1) Lesch-Nyhan syndrome is caused by an inability to catalyze which of the following

reactions?

a) Adenine to AMP

b) Adenosine to AMP

c) Guanine to GMP

d) Guanosine to GMP

e) Thymine to TMP

f) Thymidine to TMP

13.2) Which gene is defective in patients with Lesch-Nyhan syndrome?

a) Multiple

b) Adenosine deaminase

c) Purine nucleoside phosphorylase

d) Hypoxanthine-guanine phosphoribosyltransferase

e) UMP synthase

42 – Intertissue Relationships in the Metabolism of Amino Acids

1) Which of the following is most responsible for affecting the size and turnover rate of

the free amino acid pool?

a) Heart

b) Brain

c) Liver

d) Kidneys

e) Skeletal muscle

2.1) Which of the following profiles indicated next would occur within two hours after

eating a meal very high in protein and low in carbohydrates?

Page 95: Quiz Biochemistry

Biochemistry 14Mar2009

DO NOT DISTRIBUTE - 95 -

Blood glucagon Liver gluconeogenesis BCAA oxidation in muscle

a) Down Down Up

b) Up Down Up

c) Down Up Up

d) Up Up Up

e) Down Down Down

2.2) During the fasting state, all of the following provide amino acids to the liver

EXCEPT:

a) Skeletal muscle

b) Brain

c) RBCs

d) Gut

e) Kidneys

3) In the postabsorptive state, glucagon ____ the activity of amino acid degradation and

____ the activity of gluconeogenesis.

a) Increases; Increases

b) Decreases; Decrease

c) Increases; Decreases

d) Decreases; Increases

4) Which of the following is regulated by glutamate in maintaining blood pH?

a) HCO3-

b) NH4+

c) H+

d) Na+

e) HPO4-

5) Glutamine is used as fuel by the kidney in the normal fed state and more importantly

during fasting and metabolic acidosis. Glycogen from the kidney is mostly used:

a) In the brain

b) By RBCs

c) In the liver

d) By the heart

e) In the renal medulla

6.1) The signal that indicates to muscle that protein degradation needs to be initiated is

which of the following?

a) Insulin

b) Glucagon

c) Epinepherine

d) Cortisol

e) Glucose

6.2) Branched chain amino acids from skeletal muscle are transported to the liver as what

amino acid?

a) Tyrosine

b) Serine

c) Alanine

d) Cysteine

e) Aspartate

Page 96: Quiz Biochemistry

Biochemistry 14Mar2009

DO NOT DISTRIBUTE - 96 -

7) During fasting, which of the following is a major fuel of the gut?

a) Glucose

b) Alanine

c) Aspartate

d) Glutamate

e) Glutamine

8.1) The gut uses glutamine as an energy source, but can also secrete citrulline,

synthesized from the carbons of glutamine. Which of the following compounds is an

obligatory intermediate in the conversion (consider only the carbon atoms of glutamine

while answering the question)?

a) Aspartate

b) Succinyl CoA

c) Glutamate

d) Serine

e) Fumerate

8.2) The skeletal muscles convert BCAA carbons to glutamine for export to the rest of

the body. An obligatory intermediate, which carries carbons originally from the BCAA,

in the conversion of BCAA to glutamine, is which of the following?

a) Urea

b) Pyruvate

c) Lactate

d) Isocitrate

e) Phosphoenolpyruvate

8.3) What is the role of glutamine in the brain?

a) Nitrogen transporter

b) Neurotransmitter synthesis

c) Neurotransmitter degradation

d) BCAA formation

e) A and B

f) C and D

9) Which of the following is NOT true regarding amino acid metabolism during a

hypercatabolic state (surgery, burns, trauma, stress)?

a) Decreased protein synthesis in skeletal muscle

b) Positive nitrogen balance

c) Accelerated protein turnover rate

d) Protein degradation increased

e) Oxidation of BCAA increased

James Lamberg

Page 97: Quiz Biochemistry

Biochemistry 14Mar2009

DO NOT DISTRIBUTE - 97 -

AnswerKey

BioC #1

1) A

2.1) D

2.2) B

3.1) D

3.2) C

3.3) A

3.4) E

4) D

5.1) C

5.2) C

6) E

7.1) D

7.2) A

8) E

9) B

10) C

11.1) D

11.2) B

12) B

13) A

14) E

15) A

16) D

17) C

18) E

19.1) C

19.2) D

19.3) B

20.1) E

20.2) C

20.3) D

20.4) C

20.5) B

21) B

22) D

BioC #2

1.1) E

1.2) C

1.3) A

1.4) D

1.5) B

1.6) E

2) B

3.1) A

3.2) D

3.3) C

4.1) D

4.2) B

5.1) A

5.2) E

BioC #3

1.1) A

1.2) E

1.3) A

1.4) D

2) C

3.1) E

3.2) B

3.3) A

4) D

5) C

6) A

BioC #4

1.1) C

1.2) D

1.3) A

2) D

3.1) E

3.2) C

4.1) C

4.2) A

5) E

6) A

7) D

8.1) C

8.2) A

8.3) E

8.4) C

BioC #5

1) D

2.1) A

2.2) C

2.3) E

2.4) B

2.5) D

3) C

4) C

5) A

6) B

7) E

8) D

9) C

10) E

11) C

12) D

13) B

BioC #6

1.1) B

1.2) C

1.3) E

2.1) D

2.2) A

2.3) A

2.4) E

3.1) B

3.2) E

3.3) C

4.1) A

4.2) B

5.1) B

5.2) D

5.3) A

5.4) C

5.5) B

5.6) E

6) E

7) B

8) D

9) C

10) E

11) B

BioC #7

1) E

2.1) B

2.2) B

3.1) E

3.2) D

4) B

5) E

6.1) C

6.2) D

6.3) A

7) E

8.1) B

8.2) C

9) D

10) A

11) E

12) A

13) B

14.1) D

14.2) A

BioC #8

1) C

2) B

3.1) D

3.2) C

3.3) C

3.4) E

3.) A

3.6) B

4.1) A

4.2) C

5.1) D

5.2) C

5.3) A

5.4) D

5.5) B

6.1) C

6.2) B

6.3) A

7) C

BioC #9

1.1) B

1.2) A

1.3) C

2) C

3.1) E

3.2) C

4.1) C

4.2) A

5) E

6.1) C

6.2) D

Page 98: Quiz Biochemistry

Biochemistry 14Mar2009

DO NOT DISTRIBUTE - 98 -

7) D

8) B

9) C

BioC #10

1.1) D

1.2) D

2) A

3) E

4.1) D

4.2) B

5) C

6) A

7.1) D

7.2) E

BioC #11

1.1) C

1.2) D

2) B

3) D

4) E

5) B

6.1) C

6.2) D

7) A

8) C

9) B

10.1) E

10.2) D

11.1) A

11.2) C

11.3) E

12) E

13) B

14) B

BioC #12

1.1) E

1.2) A

1.3) B

2.1) C

2.2) D

3) C

4.1) D

4.2) B

5) E

6) C

7.1) D

7.2) B

BioC #13

1) D

2) E

3.1) B

3.2) A

3.3) C

4) C

5) D

BioC #14

1.1) A

1.2) B

2.1) B

2.2) B

2.3) C

2.4) D

2.5) C

2.6) A

3) E

4.1) D

4.2) C

4.3) B

5.1) D

5.2) C

5.3) C

BioC #15

1) C

2) B

3.1) D

3.2) B

3.3) C

3.4) A

3.5) E

4.1) D

4.2) B

5.1) A

5.2) C

5.3) B

5.4) E

6) D

7) D

8.1) D

8.2) B

8.3) C

8.4) A

BioC #16

1) E

2.1) C

2.2) D

2.3) A

3.1) E

3.2) C

3.3) A

BioC #19

1.1) D

1.2) E

1.3) C

2) E

3) D

4) B

5) A

6) D

7.1) C

7.2) E

BioC #20

1.1) D

1.2) B

1.3) D

2) E

3.1) E

3.2) B

3.3) A

3.4) C

3.5) E

4) E

5.1) B

5.2) D

5.3) C

5.4) E

6) A

7.1) B

7.2) B

7.3) C

7.4) C

8) E

BioC #21

1.1) E

1.2) C

2.1) C

2.2) D

2.3) E

2.4) A

3) C

4.1) B

4.2) D

4.3) A

4.4) D

5.1) E

5.2) C

6) C

7) E

BioC #22

1.1) B

1.2) D

1.3) H

1.4) B

1.5) D

1.6) B

2) C

3) C

4) A

5) E

6) B

BioC #23

1.1) C

1.2) B

1.3) D

2.1) E

2.2) A

2.3) E

3.1) D

3.2) B

4) C

5.1) D

5.2) E

5.3) C

Page 99: Quiz Biochemistry

Biochemistry 14Mar2009

DO NOT DISTRIBUTE - 99 -

6) B

7.1) D

7.2) B

7.3) C

7.4) A

8) C

9) C

BioC #24

1) C

2) A

3.1) D

3.2) B

4.1) E

4.2) B

5.1) E

5.2) E

6.1) B

6.2) A

7.1) D

7.2) C

7.3) D

7.4) C

BioC #25

1.1) B

1.2) A

2.1) A

2.2) E

3) D

4) E

5.1) D

5.2) B

5.3) C

6) E

7) A

BioC #26

1.1) A

1.2) D

2.1) A

2.2) D

2.3) C

3) B

4) E

5.1) E

5.2) B

5.3) A

5.4) C

BioC #27

1.1) A

1.2) B

1.3) E

2.1) A

2.2) D

3) C

4.1) A

4.2) B

5.1) C

5.2) D

6.1) E

6.2) A

6.3) C

7) D

BioC #28

1) A

2) C

3.1) D

3.2) A

3.3) E

3.4) B

4.1) E

4.2) B

5.1) D

5.2) C

5.3) C

6.1) C

6.2) A

7.1) B

7.2) C

BioC #29

1.1) E

1.2) C

2.1) C

2.2) D

3.1) B

3.2) E

4) A

5) C

6) A

7) E

BioC #30

1.1) E

1.2) D

1.3) C

1.4) C

2.1) B

2.2) E

2.3) D

3) A

4) A

5.1) C

5.2) A

5.3) B

6.1) A

6.2) A

6.3) E

7.1) D

7.2) A

8) D

9.1) C

9.2) C

10.1) B

10.2) E

11) A

12.1) A

12.2) B

BioC #31

1.1) D

1.2) A

1.3) C

1.4) B

1.5) D

1.6) C

2) B

3) A

4) D

5) E

6.1) A

6.2) A

7) B

8) C

9) B

10) E

11) A

12) B

13) D

14) C

15.1) E

15.2) A

16) B

17) D

BioC #32

1) C

2) C

3) B

4.1) B

4.2) C

4.3) E

5.1) D

5.2) A

5.3) E

6.1) B

6.2) A

6.3) D

6.4) C

BioC #33

1.1) A

1.2) E

1.3) C

2) B

3.1) B

3.2) B

4.1) D

4.2) A

4.3) C

5) E

6) D

7.1) E

7.2) B

7.3) A

8.1) E

8.2) E

9.1) C

9.2) B

9.3) A

9.4) D

Page 100: Quiz Biochemistry

Biochemistry 14Mar2009

DO NOT DISTRIBUTE - 100 -

10) E

11.1) C

11.2) B

11.3) D

11.4) B

11.5) A

11.6) C

12) A

BioC #34

1) A

2.1) C

2.2) D

2.3) B

3.1) B

3.2) E

4) B

5) D

6.1) B

6.2) B

6.3) D

7.1) B

7.2) E

8.1) B

8.2) A

9) D

10.1) C

10.2) D

11) E

12) D

BioC #35

1.1) B

1.2) C

1.3) A

1.4) C

1.5) E

2.1) A

2.2) A

2.3) D

3.1) C

3.2) D

4) C

5.1) C

5.2) B

5.3) A

5.4) E

BioC #36

1.1) B

1.2) D

1.3) B

1.4) A

1.5) E

2) B

3.1) E

3.2) C

4) D

BioC #37

1) D

2) C

3.1) C

3.2) C

3.3) B

3.4) A

4.1) D

4.2) E

4.3) C

4.4) B

5) D

6) C

7.1) A

7.2) D

8) E

9) A

BioC #38

1.1) A

1.2) C

2.1) D

2.2) A

3) D

4) C

5) B

6) E

7.1) C

7.2) D

8) A

9) B

10) D

11) C

12) E

13) A

14.1) C

14.2) B

14.3) D

15) E

16) B

17) D

18.1) A

18.2) C

18.3) E

18.4) B

18.5) D

19) A

BioC #39

1.1) A

1.2) E

2.1) E

2.2) C

2.3) D

3.1) C

3.2) B

3.3) D

4) A

5) B

6) C

7) C

8.1) A

8.2) E

9) D

10) C

11) B

BioC #40

1.1) B

1.2) C

2.1) B

2.2) E

3) A

4) D

5) B

6) B

7.1) E

7.2) A

7.3) B

7.4) D

7.5) C

8) E

9) D

10.1) E

10.2) D

11.1) C

11.2) B

11.3) A

12) C

13) C

14) E

BioC #41

1) B

2) C

3) E

4) B

5) A

6) D

7) D

8) C

9.1) A

9.2) D

10) E

11) B

12.1) B

12.2) B

12.3) A

13.1) C

13.2) D

BioC #42

1) E

2.1) D

2.2) C

3) A

4) B

5) E

6.1) D

6.2) C

7) E

8.1) C

8.2) D

8.3) E

9) B